You are on page 1of 71

©Educational Training Services, A Division of MLI Consulting, Inc.

GMAT DATA SUFFICIENCY TESTS 1-6 WITH EXPLANATIONS


THIS FILE IS MEANT FOR USE BY PARTICIPANTS REGISTERED IN OUR PREP COURSES

DATA SUFFICIENCY TESTS 1


THROUGH 6 WITH
EXPLANATIONS

DIRECTIONS: This file includes 6 tests, each containing about 20


questions, that need to be completed by you. Try to complete each test
within the allotted time indicated at the beginning of the test.

Start timing yourself at the beginning of the test and indicate the time
taken to complete each test at the end of each test.

1
©Educational Training Services, A Division of MLI Consulting, Inc.
GMAT DATA SUFFICIENCY TESTS 1-6 WITH EXPLANATIONS
THIS FILE IS MEANT FOR USE BY PARTICIPANTS REGISTERED IN OUR PREP COURSES
4. Last Friday a certain shop sold 3/4 of its
inventory of sweaters. Each sweater sold for
DATA SUFFICIENCY $20. What was the total revenue last Friday
EXERCISE 1 from the sale of these sweaters?

(1) When the shop opened last Friday, there


were 160 sweaters in the inventory.
CHOOSE: (2) All but 40 sweaters in the shop’s inventory
A Statement (1) alone is sufficient but statement were sold last Friday
(2) alone is not sufficient
B Statement (2) alone is sufficient but
Your Choice: A ( ) B ( ) C ( ) D ( ) E ( )
statement (1) alone is not sufficient.
C BOTH statement TOGETHER are
sufficient, but NEITHER statement ALONE IS 5. A jar containing 30 marbles, of which 20 are
sufficient. red and 10 are blue. If 9 of the marbles are
D EACH statement ALONE is sufficient removed, how many of the marbles left in the jar
are red?
E Statements (1) and (2) TOGETHER are NOT
sufficient.
(1) Of the marbles removed, the ratio of the
number of red ones to the number of blue ones is
1. In the figure below, is CD > BC ? 2:1.
(2) Of the first 6 marbles removed, 4 are red.
A B C D
Your Choice: A ( ) B ( ) C ( ) D ( ) E ( )
(1) AD=20 (2) AB = CD
6. Is the triangle ABC, whose angles are x,y and
z equilateral?
Your Choice: A ( ) B ( ) C ( ) D ( ) E ( )
(1) x = y
2. How many more men than women are in the (2) z=60 degrees
room?
Your Choice: A ( ) B ( ) C ( ) D ( ) E ( )
(1) There is a total of 20 women and men in the
room.
(2) The number of men in the room equals the 7. If w + z = 28, what is the value of wz
square of the number of women in the room.
(1) w and z are positive integers
(2) w and z are consecutive odd integers
Your Choice: A ( ) B ( ) C ( ) D ( ) E ( )
Your Choice: A ( ) B ( ) C ( ) D ( ) E ( )
3. If n is an integer, is (100-n) an integer?
n 8. Is ax = 3 - bx ?
(1) n > 4
(2) n2 = 25 (1) x (a+b) = 3
(2) a = b = 1.5 and x = 1
Your Choice: A ( ) B ( ) C ( ) D ( ) E ( )
Your Choice: A ( ) B ( ) C ( ) D ( ) E ( )

2
©Educational Training Services, A Division of MLI Consulting, Inc.
GMAT DATA SUFFICIENCY TESTS 1-6 WITH EXPLANATIONS
THIS FILE IS MEANT FOR USE BY PARTICIPANTS REGISTERED IN OUR PREP COURSES
9. What is the value of integer x ? 14. What is the average (arithmetic mean) of j
and k?
(1) x is a prime number
(2) 30 < x < 38 (1) The average of j + 2 and k + 4 is 11
(2) The average of j, k, and 14 is 10.
Your Choice: A ( ) B ( ) C ( ) D ( ) E ( )
Your Choice: A ( ) B ( ) C ( ) D ( ) E ( )

10. While on a straight road, car X and Car Y are


traveling at different constant rates. If car X is 15. What is the value of a - b?
now 1 mile ahead of car Y. How many minutes
from now will car X be 2 miles ahead of car Y? (1) a = b + 4
(2) (a - b)2 = 16
(1) Car X is traveling at 50 miles per hour and
car Y is traveling at 40 miles per hour. Your Choice: A ( ) B ( ) C ( ) D ( ) E ( )
(2) 3 minutes ago car X was 1/2 mile ahead of
car Y. 16. Is rst = 1?

Your Choice: A ( ) B ( ) C ( ) D ( ) E ( ) (1) rs = 1


(2) st = 1
11. In what year was Ellen born?
Your Choice: A ( ) B ( ) C ( ) D ( ) E ( )
(1) Ellen’s brother Pete, who is 1 1/2 years older
than Ellen, was born in 1956. 17. In a certain office, 50 percent of the
(2) In 1975 Ellen turned 18 years old. employees are college graduates and 60 percent
of the employees are over 40 years old. If 30
Your Choice: A ( ) B ( ) C ( ) D ( ) E ( ) percent of those over forty have master’s
degrees, how many of the employees over forty
12. What is the number of female employees in have master’s degree?
Company X?
(1) Exactly 100 of the employees are college
(1) If company X were to hire 14 more people graduates.
and all of these people were females, the ratio of (2) Of the employees forty years old or less, 25
the number of male employees to the number of percent have master’s degrees.
female employees would then be 16 to 9.
(2) Company X has 105 more male employees Your Choice: A ( ) B ( ) C ( ) D ( ) E ( )
than female employees.
18. Is xy < 6 ?
Your Choice: A ( ) B ( ) C ( ) D ( ) E ( )
(1) x < 3 and y < 2
(2) 1/2 < x < 2/3 and y2< 64
13. Is the integer x divisible by 36?
Your Choice: A ( ) B ( ) C ( ) D ( ) E ( )
(1) x is divisible by 12
(2) x is divisible by 9.

Your Choice: A ( ) B ( ) C ( ) D ( ) E ( )

3
©Educational Training Services, A Division of MLI Consulting, Inc.
GMAT DATA SUFFICIENCY TESTS 1-6 WITH EXPLANATIONS
THIS FILE IS MEANT FOR USE BY PARTICIPANTS REGISTERED IN OUR PREP COURSES
19. Is x equal to 2?

(1) x, y and x + y are prime numbers


(2) y is odd

Your Choice: A ( ) B ( ) C ( ) D ( ) E ( )

20. In 1979 Mr. Jackson bought a total of n


shares of stock X and Mrs. Jackson bought a
total of 300 shares of stock X. If the couple held
all of their respective shares throughout 1980,
and Mr. Jackson’s 1980 dividends on his n
shares totaled $150, what was the total amount
of Mrs. Jackson’s 1980 dividends on her 300
shares ?

(1) In 1980 the annual dividend on each share of


stock X was $0.75
(2) In 1979, Mr. Jackson bought a total of 200
shares of stock X.

Your Choice: A ( ) B ( ) C ( ) D ( ) E ( )

End of Section Test. If you took longer than 25


minutes to complete, record that information
here: Additional Minutes.

4
©Educational Training Services, A Division of MLI Consulting, Inc.
GMAT DATA SUFFICIENCY TESTS 1-6 WITH EXPLANATIONS
THIS FILE IS MEANT FOR USE BY PARTICIPANTS REGISTERED IN OUR PREP COURSES

DATA SUFFICIENCY 4. If n is a member of the set


EXERCISE 2 (33, 36, 38, 39, 41, 42),
20 Questions -- 25 Minutes What is the value of n?
CHOOSE:
A Statement (1) alone is sufficient but statement (1) n is even
(2) alone is not sufficient
B Statement (2) alone is sufficient but (2) n is a multiple of 3
statement (1) alone is not sufficient.
C BOTH statement TOGETHER are A( ) B( ) C( ) D( ) E( )
sufficient, but NEITHER statement ALONE IS
sufficient. 5. What is the value of x?
D EACH statement ALONE is sufficient
E Statements (1) and (2) TOGETHER are NOT (1) 2x + 1 = 0
sufficient.
(2) (x + 1 )² = x²
1. If the list price of a new car was $12,300,
what was the cost of the car to the dealer? A( ) B( ) C( ) D( ) E( )

(1) The cost to the dealer was equal to 80 percent 6. In the figure below, what is the length of AD?
of the list price
A_________B_________C_________D
(2) The car was sold for $11,070, which was
12.5 percent more than the cost to the dealer. (1) AC = 6

A( ) B( ) C( ) D( ) E( ) (2) BD = 6

2. If p, q, x, y, and z are different positive A( ) B( ) C( ) D( ) E( )


integers, which of the five integers is the
median? 7. A retailer purchased a television set for x
percent less than its list price, and then sold it
(1) p + x < q for y percent less than its list price. What was
the list price of the television set?
(2) y < z
(1) x = 15
A( ) B( ) C( ) D( ) E( )
(2) x - y = 5
3. A certain employee is paid $6 per hour for an
8-hour workday. If the employee is paid 1 1/2 A( ) B( ) C( ) D( ) E( )
times this rate for time worked in excess of 8
hours during a single day, how many hours did
the employee work today? 8. Is x ² greater than x ?
(1) The employee was paid $18 more for hours
worked today than for hours worked yesterday.
²
(1) x is greater than 1
(2) x is greater than -1
(2) Yesterday the employee worked 8 hours
A( ) B( ) C( ) D( ) E( )
A( ) B( ) C( ) D( ) E( )

5
©Educational Training Services, A Division of MLI Consulting, Inc.
GMAT DATA SUFFICIENCY TESTS 1-6 WITH EXPLANATIONS
THIS FILE IS MEANT FOR USE BY PARTICIPANTS REGISTERED IN OUR PREP COURSES
9. What is the value of r/2 + s/2 ?

(1) (r + s)/2 =5 12. In 1979 Mr. Jackson bought a total of n


shares of stock X and Mrs. Jackson bought a
(2) r + s = 10 total of 300 shares of stock X. If the couple held
all of their respective stocks throughout 1980,
A( ) B( ) C( ) D( ) E( ) and Mr. Jackson’s 1980 dividends on his n
shares totaled $150, what was the total amount
10. If x, y, and z are numbers, is z = 18 ? of Mrs. Jackson’s 1980 dividends on her 300
shares ?
(1) The average (arithmetic mean) of x, y, and z
is 6 (1) In 1980 the annual dividend on each share of
stock X was $ 0.75
(2) x = - y
(2) In 1979 Mr. Jackson bought a total of 200
A( ) B( ) C( ) D( ) E( ) shares of stock X

11. A( ) B( ) C( ) D( ) E( )

B 13. If Sara’s age is exactly twice Bill’s age, what


is Sara’s age?
C
(1) Four years ago, Sara’s age was exactly 3
A times Bill’s age.

The circular base of an above ground swimming (2) Eight years from now, Sara’s age will be
pool lies in a level yard and just touches two exactly 1.5 times Bill’s age.
straight sides of a fence at points A and B, as
shown in the figure above. Point C is on the A( ) B( ) C( ) D( ) E( )
ground where the two sides of the fence meet.
How far from the center of the pool’s base is 14. What is the value of (x/yz)?
point A?
(1) x = y/2 and z = 2x/5
(1) The base has area 250 square feet
(2) x/z = 5/2 and 1/y = 1/10
(2) The center of the base is 20 feet from point
C. A( ) B( ) C( ) D( ) E( )

A( ) B( ) C( ) D( ) E( ) 15. An infinite sequence of positive integers is


called an “alpha sequence” if the number of even
integers in the sequence is finite. If S is an
infinite sequence of positive integers, is S an
alpha sequence?

(1) The first ten integers in S are even.

(2) An infinite number of integers in S are odd.

A( ) B( ) C( ) D( ) E( )

6
©Educational Training Services, A Division of MLI Consulting, Inc.
GMAT DATA SUFFICIENCY TESTS 1-6 WITH EXPLANATIONS
THIS FILE IS MEANT FOR USE BY PARTICIPANTS REGISTERED IN OUR PREP COURSES
16. If xy > 0, does (x-1)(y-1) = 1 ? 19. Cancellation Fees Schedule

(1) x + y = xy Days prior to Percent of


Departure Package Price
(2) x = y
46 or more 10%
A( ) B( ) C( ) D( ) E( )
45 - 31 35%
17. After winning 50 percent of the first 20 30 - 16 50%
games it played, Team A won all of the 15 - 5 65%
remaining games. What was the total number of 4 or fewer 100%
games that Team A won?
The table above shows the cancellation fee
(1) Team A played 25 games altogether. schedule that a travel agency uses to determine
the fee charged to a tourist who cancels a trip
(2) Team A won 60 percent of all the games it prior to departure. If a tourist canceled a trip
played. with a package price of $1,700 and a departure
date of September 4, on what day was the trip
A( ) B( ) C( ) D( ) E( ) canceled?
18. @ + $ = * (1) The cancellation fee was $595
In the addition problem above, each of the (2) If the trip had been canceled one day later,
symbols @, $, and * represents a positive digit. the cancellation fee would have been $255 more.
If @ < $, what is the value of $?
A( ) B( ) C( ) D( ) E( )
(1) * = 4
20. Is 5k less than 1,000?
(2) @ = 1
(1) 5k+1 is greater than 3,000
A( ) B( ) C( ) D( ) E( )
(2) 5k-1 is 500 less than 5k.

A( ) B( ) C( ) D( ) E( )

END OF SECTION END OF SECTION

Did you complete the section in 25 Minutes?


If not, indicate how much longer you took to
complete this section: Additional Minutes.

7
©Educational Training Services, A Division of MLI Consulting, Inc.
GMAT DATA SUFFICIENCY TESTS 1-6 WITH EXPLANATIONS
THIS FILE IS MEANT FOR USE BY PARTICIPANTS REGISTERED IN OUR PREP COURSES
DATA SUFFICIENCY
EXERCISE 3 4. If x is an integer, is y an integer?

21 Questions -- 27 Minutes (1) The average (arithmetic mean) of x, y, and


y - 2 is x .
CHOOSE:
A Statement (1) alone is sufficient (to give a (2) y - x = 1
unique value for the information sought) but
statement (2) alone is not sufficient A ( ) B( )C( ) D( ) E ( )
B Statement (2) alone is sufficient but
statement (1) alone is not sufficient. 5. How many minutes does it take for a circular
C BOTH statement TOGETHER are shaped illumination to quadruple its area?
sufficient, but NEITHER statement ALONE IS
sufficient. (1) The initial diameter of the circular shaped
D EACH statement ALONE is sufficient illumination is 10 in.
E Statements (1) and (2), alone or
TOGETHER, are NOT sufficient. (2) The circumference of the circular shaped
illumination increases at the rate of 6 in/minute
1. The results of a certain experiment included 6
A ( ) B( )C( ) D( ) E ( )
data values that were all multiples of the same
number c, namely, c, 8c, 2c, 5c, 4c, and 4c. Was
6. The inside of a rectangular carton is 48
the average (arithmetic mean) of the 6 data
centimeters long, 32 centimeters wide, and 15
values greater than 8?
centimeters high. The carton is filled to capacity
(1) c < 4
with k identical cylindrical cans of fruit that
(2) c > 2
stand upright in rows and columns as shown
below. If the cans are 15 centimeters high, what
is the value of k?
A ( ) B( )C( ) D( ) E ( )

2. If n is an integer, is n + 2 a prime number?

(1) n is a prime number


(1) Each of the cans has a radius of 4
(2) 30 < n < 40
centimeters.
A ( ) B( )C( ) D( ) E ( )
(2) 6 of the cans fit along the length of the
carton.
3. If t is not equal to zero, is r a positive
number?
A ( ) B( )C( ) D( ) E ( )
(1) rt = 12
7. If k is a positive integer, is the value of
(b – a) at least twice the value of 3k - 2k?
(2) r + t = 7

A ( ) B( )C( ) D( ) E ( ) (1) a is equal to 2k+1 and b is equal to 3k+1


(2) k = 3

A ( ) B( )C( ) D( ) E ( )

8
©Educational Training Services, A Division of MLI Consulting, Inc.
GMAT DATA SUFFICIENCY TESTS 1-6 WITH EXPLANATIONS
THIS FILE IS MEANT FOR USE BY PARTICIPANTS REGISTERED IN OUR PREP COURSES
8. If P and Q are each circular regions, what is 13. A ladder is propped up against a vertical
the radius of the larger of these regions? wall such that the bottom of the ladder makes an
angle of 60 degrees to the ground. The ladder
(1) The area of P plus the area of Q is equal to slips down and the bottom of the ladder moves
90 pi. away from the wall. In this position, the bottom
of the ladder makes an angle of 45 deg rees to
(2) The larger circular region has a radius that is the ground. How much farther did the ladder
3 times that of the smaller circular region. move from the wall when it slipped?

A ( ) B( )C( ) D( ) E ( ) (1) The ladder is 10 meters long

9. Is a+ b divisible by 8? (2) The bottom of the ladder was 5 meters from


the wall in its original position.
(1) When a is divided by 8, it leaves a
remainder of 3. A ( ) B( )C( ) D( ) E ( )

(2) When b is divided by 8, it leaves a remainder 14. Can the positive integer n be written as the
of 5. sum of two different positive prime numbers?

A ( ) B( )C( ) D( ) E ( ) (1) n is greater than 3

10. What is the ratio of x : y : z? (2) n is odd.

(1) z = 1 and xy = 32 A ( ) B( )C( ) D( ) E ( )

(2) x = 2y and y = 4z 15. What is the last number in a set comprising


six consecutive odd integers?
A ( ) B( )C( ) D( ) E ( )
(1) The average (arithmetic mean) of the six
11. If 3m = 5n , what is the value of m + n ? consecutive odd integers is 36.

(1) 2m + n = 26 (2) Twice the average (arithmetic mean) of the


six consecutive odd integers is equal to the sum
(2) If the value of m were increased by 2, then of the first and the last number in the set.
it will be twice the value of n.
A ( ) B( )C( ) D( ) E ( )
A ( ) B( )C( ) D( ) E ( )
16. Town T has 20,000 residents, 60 percent of
12. On a Friday Morning, a certain machine ran whom are female. What percent of the residents
continuously at a uniform rate to till a were born in Town T?
production order. At what time did it completely
fill the order that morning ? (1) The number of female residents who were
born in Town T is twice the number of male
(1) The machine began filling the order at 9:30 residents who were not born in Town T.
a.m.
(2) The number of female residents who were
(2) The machine had filled 1/2 of the order by not born in Town T is twice the number of
10:30 a.m. and 5/6 of the order by 11:10 a.m. female residents who were born in Town T.

A ( ) B( )C( ) D( ) E ( ) A ( ) B( )C( ) D( ) E ( )

9
©Educational Training Services, A Division of MLI Consulting, Inc.
GMAT DATA SUFFICIENCY TESTS 1-6 WITH EXPLANATIONS
THIS FILE IS MEANT FOR USE BY PARTICIPANTS REGISTERED IN OUR PREP COURSES
17. A right triangular region PQR lies in a
rectangular coordinate plane such that each of its
sides, PQ and PR, lies parallel to the rectangular 21. Is the positive integer n equal to the square
coordinate axes. Is the right triangle isosceles? of an integer ?

(1) One of the points Q on the hypotenuse RQ (1) For every prime number p, if p is a divisor
has the coordinates (3, 4). of n, then so is p-squared..

(2) One of the points R on the hypotenuse RQ (2) Square root of n is an integer.
has the coordinates (-2, -1)
A ( ) B( )C( ) D( ) E ( )
A ( ) B( )C( ) D( ) E ( )
Did you complete this section in 25 Minutes ?
18. If both x and y are non-zero numbers, what If not, indicate how much longer did you take
is the value of y/x? to complete it ? Minutes.

(1) x = 8

(2) y³ = x²

A ( ) B( )C( ) D( ) E ( )

19. If x = 0.rstu where r, s, t, and u each


represent a non-zero digit of x, what is the value
of x ?

(1) r = 3s = 2t = 6u

(2) The product of r and u is equal to the


product of s and t

A ( ) B( )C( ) D( ) E ( )

20. If x and y are integers between 10 and 99,


inclusive, is (x - y) /9 an integer ?

(1) x and y have the same two digits but in the


reverse order.

(2) The tens’ digit of x is 2 more than the units’


digit and the tens’ digit of y is 2 less than the
units’ digit.

A ( ) B( )C( ) D( ) E ( )

10
©Educational Training Services, A Division of MLI Consulting, Inc.
GMAT DATA SUFFICIENCY TESTS 1-6 WITH EXPLANATIONS
THIS FILE IS MEANT FOR USE BY PARTICIPANTS REGISTERED IN OUR PREP COURSES

Data Sufficiency Exercise 4 1. Is x < y?

(1) x2 < y2
Time : 25 Minutes 20 Questions.
(2) x < y + 1
Directions: Each of the Data Sufficiency Your Answer:
problems below consists of a question and two
statements, labeled (1) and (2), in which certain 2. Machine A runs at a constant rate and
data are given. You have to decide whether the produces a lot consisting of 100 bolts in 30
data given in the statements are sufficient for minutes. How much less time would it take to
answering the question. Using the data given in produce the lot of bolts if both machines A and
the statements plus your knowledge of B were run simultaneously?
mathematics and everyday facts (such as the
number of days in July or the meaning of (1) Both machines A and B produce the same
counterclockwise), you are to fill in oval, number of bolts per hour.

A if statement (1) ALONE is sufficient, but (2) It takes machine A twice as long as it takes
statement (2) alone is not sufficient to answer the machines A and B, running simultaneously, to
question asked; produce the same lot of bolts.

B if statement (2) ALONE is sufficient, but Your Answer:


statement (1) alone is not sufficient to answer the
question asked; 3. Is p a prime number?

C if BOTH statements TOGETHER are (1) p+1 is a prime number.


sufficient to answer the question asked, but
NEITHER statement alone is sufficient. (2) p is even integer.

D if EACH statement ALONE is sufficient to Your Answer:


answer the question asked;
4. Is the hundredths digit of the decimal number
E if statement (10 and (2) TOGETHER are NOT d greater than 5?
sufficient to answer the question asked, and
additional data specific to the problem are (1) The tenths digit of the decimal number 10d
needed. is 7

All numbers used are real numbers. A figure in (2) The thousandths digit of d/10 is 7.
the data sufficiency problem will conform to the
information given in the question, but will not
necessarily conform to the additional Your Answer:
information given in statements (1) and (2).
5. If n is an integer, is (100 - n)
All figures lie in a plane unless otherwise an integer? 2n
indicated.
(1) n < 60
In questions that ask for the value of a quantity,
the data given in the statements are sufficient (2) n is divisible by 10.
only when it is possible to determine exactly one Your Answer:
numerical value for the quantity.

11
©Educational Training Services, A Division of MLI Consulting, Inc.
GMAT DATA SUFFICIENCY TESTS 1-6 WITH EXPLANATIONS
THIS FILE IS MEANT FOR USE BY PARTICIPANTS REGISTERED IN OUR PREP COURSES
6. If -10 < k < 10 , is k > 0 ?

(1) k2 > 4 11. If x is an integer, is y an integer?

(2) 1 - k > 0 (1) The average of x and y is NOT an


integer.
Your Answer:
(2) ( x + y ) = 2( x - y )
7. Buckets X and Y contained only water and
bucket Y was half full. If all the water in bucket Your Answer:
X was poured into bucket Y, what fraction of the
capacity of Y was then filled with water? 12. The price per share of stock X increased by
10 percent over the same time period that the
(1) Before the water from X was poured into Y, price per share of stock Y decreased by 10
X was 1/4 full. percent. The reduced price per share of stock Y
was what percent of the original price per share
(2) X has twice the capacity of Y. of stock X ?

Your Answer: (1) The increased price per share of stock X was
equal to the original price per share of stock Y.
8. Did Anne pay less than d dollars, including
tax, for her mink coat? (2) The increase in price per share of stock X
was 10/11 the decrease in the price per share of
(1) The price Anne paid for her mink coat was stock Y.
(0.9.d), excluding tax.
Your Answer:
(2) The tax payable on mink coat sale is 10
percent of the selling price. 13. Any decimal that has only a finite number
of non-zero digits is a terminating decimal.
Your Answer: Examples: 32, 0.78, and 6.087 are three
terminating decimals. If r and s are positive
9. Is c > d? integers, is the ratio r/s a terminating decimal?

(1) 1 - c/d > -1 (1) 90 < r < 100

(2) 0.5 < c/d < 2.0 (2) s = 3

Your Answer: Your Answer:

10. In a certain health club, are more than 2/3 14. What is the area of the rectangular region
of the members females? of sides L and W?

(1) The club has exactly 75 female members. (1) L + W = 14

(2) The ratio of female to male members is 3:1 (2) d = 10 , (d is the diagonal)

Your Answer: Your Answer:

12
©Educational Training Services, A Division of MLI Consulting, Inc.
GMAT DATA SUFFICIENCY TESTS 1-6 WITH EXPLANATIONS
THIS FILE IS MEANT FOR USE BY PARTICIPANTS REGISTERED IN OUR PREP COURSES
15. Is 2n > 100?
18. In a certain office, 60 percent of the
employees are college graduates and 50 percent
(1) 3n > 100 of the employees are over forty years old. If 30
percent of those over forty years of age have a
master’s degree, how many employees are
(2) 2n+1 = 128 college graduates?

Your Answer: (1) Exactly 60 employees are over 40 and have


a master’s degree.

16. (2) There are four times as many college


graduates as there are those over forty with
master’s degrees.

Your Answer:

19. If n is an integer, is n an odd integer?

(1) (3n + 1) is an odd integer

(2) (2n - 1) is an odd integer


Your Answer:
Your Answer:
17.
20.

Your Answer:

End of Section End of Section

Your Answer: Time you took to complete this section:


Minutes

13
©Educational Training Services, A Division of MLI Consulting, Inc.
GMAT DATA SUFFICIENCY TESTS 1-6 WITH EXPLANATIONS
THIS FILE IS MEANT FOR USE BY PARTICIPANTS REGISTERED IN OUR PREP COURSES
Data Sufficiency Exercise 5

Time : 27 Minutes 21 Questions.


1. What is the value of | n |?
Directions: Each of the Data Sufficiency
problems below consists of a question and two (1) n is a negative integer.
statements, labeled (1) and (2), in which certain
data are given. You have to decide whether the (2) n2 = 9
data given in the statements are sufficient for
answering the question. Using the data given in Your Answer:
the statements plus your knowledge of
mathematics and everyday facts (such as the 2. If x, y and z are the three internal angles of
number of days in July or the meaning of a triangle, what is the value, in degrees, of angle
counterclockwise), you are to fill in oval, x ? (all values of x, y and z are in degrees)

A if statement (1) ALONE is sufficient, but (1) x + y = 127


statement (2) alone is not sufficient to answer the
question asked; (2) x + y = 2z + 21

B if statement (2) ALONE is sufficient, but Your Answer:


statement (1) alone is not sufficient to answer the
question asked; 3. Of the n people participating in a test
preparation program, 60 percent had not taken
C if BOTH statements TOGETHER are the test previously. Of the remaining, 25 percent
sufficient to answer the question asked, but had taken the test more than once previously.
NEITHER statement alone is sufficient. How many had taken the test just once
previously?
D if EACH statement ALONE is sufficient to
answer the question asked; (1) 12 people had taken the test more than twice
previously.
E if statement (10 and (2) TOGETHER are NOT
sufficient to answer the question asked, and (2) 18 people had taken the test more than once
additional data specific to the problem are previously.
needed.
Your Answer:
All numbers used are real numbers. A figure in
the data sufficiency problem will conform to the 4. How many more men than women are in the
information given in the question, but will not public swimming pool?
necessarily conform to the additional
information given in statements (1) and (2). (1) There are twice as many men as there are
women.
All figures lie in a plane unless otherwise
indicated. (2) If six more women came into the pool, there
will be an equal number of men and women.
In questions that ask for the value of a quantity,
the data given in the statements are sufficient Your Answer:
only when it is possible to determine exactly one
numerical value for the quantity.

14
©Educational Training Services, A Division of MLI Consulting, Inc.
GMAT DATA SUFFICIENCY TESTS 1-6 WITH EXPLANATIONS
THIS FILE IS MEANT FOR USE BY PARTICIPANTS REGISTERED IN OUR PREP COURSES

5. Is the sum of a set of six consecutive odd 9. If an item was marked up 30 percent on its
positive integers a perfect cube? cost and then sold at a discount on the sticker
price, what was the selling price, in dollars?
(1) The smallest number in the set is a prime
number. (1) The item cost $120

(2) The largest number in the set is 41. (2) The sticker price was $156

Your Answer: Your Answer:

6. How many miles long is the route from


Thunder Bay to Sarnia? 10. If n is an integer, is n/15 an integer ?

(1) It will take 2 hours less time to travel the (1) 3n/15 is an integer
entire route at an average rate of 60 miles per
hour than at an average rate of 50 miles per hour. (2) 7n/15 is an integer

(2) The first half of the distance can be traveled Your Answer:
in 5 hours at 60 miles per hour rate of speed.
11. Mary and Martha received wage increases
Your Answer: following their annual performance review?
Who received the greater increase, in dollars?

7. If x is divisible by 2, is x + y an even (1) Mary received a 10 percent increase on her


number? wages.

(1) y is a factor of x (2) Martha received a 8 percent increase on her


wages.
(2) x = 6

Your Answer: Your Answer:

12. Is John ahead of Paul in the line-up?


8. All applicants to a military recruitment
program must pass both a written test and a (1) There are 20 people in the line-up.
physical test. If 60 percent of the applicants
passed the written test and 75 percent of the (2) There are exactly 6 people between Paul and
applicants passed the physical test, what percent John in the line-up.
of the applicants did not pass both or either
tests? Your Answer:

(1) 55 percent of the applicants passed both tests.

(2) 20 percent of the applicants did not pass


either test.

Your Answer:

15
©Educational Training Services, A Division of MLI Consulting, Inc.
GMAT DATA SUFFICIENCY TESTS 1-6 WITH EXPLANATIONS
THIS FILE IS MEANT FOR USE BY PARTICIPANTS REGISTERED IN OUR PREP COURSES
17. The symbol @ represents one of the
13. Lorraine can drive from her home to a following operations: addition, subtraction,
super-market by one of two possible routes. If multiplication, or division. What is the value of
she must also return by one of the two routes, 4@3?
what is the distance of the shorter route?
(1) 0 @ 4 = 4
(1) When Lorraine drives to the supermarket by
the shorter route and returns by the longer route, (2) 4 @ 0 = 4
she travels a total of 11 miles.
Your Answer:
(2) When she drives both ways, from her home
to the super-market, by the shorter route, she 18. If y = 2x+1, what is the value of y - x?
travels a total distance of 8 miles.
(1) 22x+4 = 64
Your Answer:

14. If p < 100, is the prime number p equal to (2) y = 22x


31?
Your Answer:
(1) p = n3 + 4 , where n is an integer.
19. In a certain group of people, the average
(2) p + 5 is a perfect square. (arithmetic mean) I.Q of the males is 128 and
of the females 136. What is the average I.Q. of
Your Answer: the people in the group?

15. What is the ratio of x to y ? (1) The group contains twice as many females
as males
(1) If the value of y were increased by 7, then
the ratio of x to y will be equal to 1. (2) The group contains 10 more females than
males.
(2) The ratio of x to 4y is 3 to 5.
Your Answer:
Your Answer:
20. If x, y, and z are three integers, are they
16. If a television commercial consists of a total consecutive odd integers?
of 17,280 frames on film, how long, in minutes,
does the commercial run? (1) y - x = 2

(1) The commercial runs without interruption at (2) x + y is even


the rate of 24 frames per second.
Your Answer:
(2) It takes 6 times as long to run the film as it
takes to rewind the film, and it takes a total of 21. In triangle ABC, if AB = x, BC = x + 2, and
14 minutes to do both. AC = y, then which of the three internal angles
of the triangle has the greatest degree measure?
Your Answer:
(1) y = x + 3

(2) x = 2
Your Answer:

H
HO
H OOWWL
W LO
L ON
O NG
N GGD
DI
D ID
IDD
Y
Y O
YO U
OU
UTT
TAA
AKK
KE E T
ET
TOO
O
C
CO
C OM
O MP
M PL
PLLE
ET
E TE
T E:
E :
: MMI
M IN
INNS
S
S

16
©Educational Training Services, A Division of MLI Consulting, Inc.
GMAT DATA SUFFICIENCY TESTS 1-6 WITH EXPLANATIONS
THIS FILE IS MEANT FOR USE BY PARTICIPANTS REGISTERED IN OUR PREP COURSES
1. Is x greater than 2?
D
DA
D AT
A TA
T AS
A SU
S UF
U FFF FI
F IC
I CI
C IE
I EEN NC
N CY
CYY
EEX
E XXE ER
E RC
R CI
C IS
I SE
S EE6 6
6 2
(1) x > 4
Time : 25 Minutes 20 Questions.
(2) x is a multiple of 2
Directions: Each of the Data Sufficiency
problems below consists of a question and two Your Answer; A ( ) B ( ) C ( ) D ( ) E ( )
statements, labeled (1) and (2), in which certain
data are given. You have to decide whether the 2. In an election to the Secretary of the Club, if
data given in the statements are sufficient for each of the 1,000 members voted for either Mary
answering the question. Using the data given in or Michelle (but not both), what percent of the
the statements plus your knowledge of female members voted for Michelle?
mathematics and everyday facts (such as the
number of days in July, or the meaning of (1) Eighty percent of male members voted for
counterclockwise), you are to fill in oval, Michelle.

A if statement (1) ALONE is sufficient, but (2) Twice as many male members voted for
statement (2) alone is not sufficient to answer the Michelle as female members.
question asked;
Your Answer; A ( ) B ( ) C ( ) D ( ) E ( )
B if statement (2) ALONE is sufficient, but
statement (1) alone is not sufficient to answer the
question asked; 3. During 1987, 8.7 percent of the men in the
labor force were unemployed in June as against
C if BOTH statements TOGETHER are 8.4 percent in May of that year. If the number of
sufficient to answer the question asked, but men in the labor force was the same in both
NEITHER statement alone is sufficient. months, how many more men were unemployed
in June than in May?
D if EACH statement ALONE is sufficient to
answer the question asked; (1) The number of unemployed men in the labor
force during May was 1.68 million.
E if statement (10 and (2) TOGETHER are NOT
sufficient to answer the question asked, and (2) The total number of men in the labor force
additional data specific to the problem are was 20.0 million during the two months - May
needed. and June - of 1987.

All numbers used are real numbers. A figure in Your Answer; A ( ) B ( ) C ( ) D ( ) E ( )


the data sufficiency problem will conform to the
information given in the question, but will not 4. If the average (arithmetic mean) of 4
necessarily conform to the additional numbers is 45, how many numbers are greater
information given in statements (1) and (2). than 45?

All figures lie in a plane unless otherwise (1) Two of the numbers are 60 and 45.
indicated.
(2) One of the numbers is 25.
In questions that ask for the value of a quantity,
the data given in the statements are sufficient Your Answer; A ( ) B ( ) C ( ) D ( ) E ( )
only when it is possible to determine exactly one
numerical value for the quantity.

17
©Educational Training Services, A Division of MLI Consulting, Inc.
GMAT DATA SUFFICIENCY TESTS 1-6 WITH EXPLANATIONS
THIS FILE IS MEANT FOR USE BY PARTICIPANTS REGISTERED IN OUR PREP COURSES

9. If n > 0, is n equal to the sum of two


5. Is the value of x greater than 5? different prime numbers?

(1) x3 > 125 (1) n is equal to the square of the smallest odd
prime number.
(2) 4 < x < 6
(2) n + 2 is a prime number.
Your Answer; A ( ) B ( ) C ( ) D ( ) E ( )
Your Answer; A ( ) B ( ) C ( ) D ( ) E ( )
6. Is n + 1 a prime number?

(1) n is a product of two prime numbers.

(2) n2 + 2n + 1 = 49 10. 2/3 of the N people polled in a survey


involving 2 questions said Yes to Question 1.
Your Answer; A ( ) B ( ) C ( ) D ( ) E ( ) What fraction of the people polled did NOT say
yes to both questions?

7. A rectangular frame encloses a picture. What (1) 3/5 of those who answered YES to Question
is the length in inches of the picture? 1, answered YES to Question 2.

(1) The frame measures 24 inches by 18 inches. (2) 1/3 of those polled answered NO to Question
1.
(2) Area of the frame = area of the picture it
encloses. Your Answer; A ( ) B ( ) C ( ) D ( ) E ( )

Your Answer; A ( ) B ( ) C ( ) D ( ) E ( ) 11. If s, u, and v are positive integers, is


s > v?
8. How far apart do Jane and June live? (1) s > u

(1) The local public library is 5 miles due north s u v


of Jane’s house and 12 miles due east of June’s (2) 2 = 2 + 2
house.
Your Answer; A ( ) B ( ) C ( ) D ( ) E ( )
(2) The local school is 12 miles due west of
Jane’s house and 5 miles due south of June’s 12. What is the area of the square region?
house.
(1) The diagonal is 10 inches.
Your Answer; A ( ) B ( ) C ( ) D ( ) E ( )
(2) The perimeter is 20 \/2

Your Answer; A ( ) B ( ) C ( ) D ( ) E ( )

18
©Educational Training Services, A Division of MLI Consulting, Inc.
GMAT DATA SUFFICIENCY TESTS 1-6 WITH EXPLANATIONS
THIS FILE IS MEANT FOR USE BY PARTICIPANTS REGISTERED IN OUR PREP COURSES

13. What is the measure of an external angle of


the triangle ABC? 17. How many hours does it take for Machine A
to fill a production lot, working alone?
(1) One of the internal angles measures 72
degrees. (1) Machines A and B, operating simultaneously,
can fill the production lot in 2 hours.
(2) The triangle ABC is an isosceles triangle.
(2) Machine B, working alone, can fill the lot in
Your Answer; A ( ) B ( ) C ( ) D ( ) E ( ) 5 hours.

14. What is the ratio of the volume of cube X to Your Answer; A ( ) B ( ) C ( ) D ( ) E ( )


that of cube Y?

(1) The length of an edge of cube X is 6 inches. 18. How many students are in the school?

(2) The ratio of the surface area of cube X to that (1) 40 more than 1/3 of all the students in the
of cube Y is ¼. school are taking a science course and, of these,
1/4 are taking Physics.
Your Answer; A ( ) B ( ) C ( ) D ( ) E ( )
(2) Exactly 1/8 of all the students in the school
15. What is the value of k? are taking Physics.

(1) In the xy-coordinate system, (a,b) and (a+3, Your Answer; A ( ) B ( ) C ( ) D ( ) E ( )


b+k) are two points that lie on the line defined
by the equation x = 3y - 7.
19. Is c2 + d2 > 1?
(2) k.k = 1
Your Answer; A ( ) B ( ) C ( ) D ( ) E ( ) (1) d > 0

16. What is the average (arithmetic mean) dollar (2) c/d > 1
amount of all the paychecks that John Doe
received last year? Your Answer; A ( ) B ( ) C ( ) D ( ) E ( )

(1) Last year John Doe received 26 paychecks. 20. @, &, and * are three different positive
digits and If @ + & = * , What is the value
(2) The average of John Doe’s first thirteen of &?
checks during the year was $750. The average of
John Doe’s last 13 checks was $800. (1) * =4

Your Answer; A ( ) B ( ) C ( ) D ( ) E ( ) (2) @ = 1

Your Answer; A ( ) B ( ) C ( ) D ( ) E ( )

End of Section

How long did you take to complete this


section?

19
©Educational Training Services, A Division of MLI Consulting, Inc.
GMAT DATA SUFFICIENCY TESTS 1-6 WITH EXPLANATIONS
THIS FILE IS MEANT FOR USE BY PARTICIPANTS REGISTERED IN OUR PREP COURSES

ANSWER KEY AND


EXPLANATIONS TO THE
DATA SUFFICIENCY 1 – 6
TESTS

20
©Educational Training Services, A Division of MLI Consulting, Inc.
GMAT DATA SUFFICIENCY TESTS 1-6 WITH EXPLANATIONS
THIS FILE IS MEANT FOR USE BY PARTICIPANTS REGISTERED IN OUR PREP COURSES

2. How many more men than women are in the


DATA SUFFICIENCY room?
EXERCISE 1- Answer Key
(1) There is a total of 20 women and men in the
room.
CHOOSE: (2) The number of men in the room equals the
A Statement (1) alone is sufficient but statement square of the number of women in the room.
(2) alone is not sufficient
B Statement (2) alone is sufficient but Your Choice: A ( ) B ( ) C ( ) D ( ) E ( )
statement (1) alone is not sufficient. Statement 1 gives us a total value for the number
C BOTH statement TOGETHER are of men and women in the room. We can get a
sufficient, but NEITHER statement ALONE IS range of possible values of men and
sufficient. women:m1w19,m2w18,m3w17,......m16w4......
D EACH statement ALONE is sufficient m19w1. Therefore, we conclude that statement 1
E Statements (1) and (2) TOGETHER are NOT alone is not sufficient to answer the question.
sufficient. The choices before us are B,C, or E. Statement 2
tells us that the number of men in the room is
1. In the figure below, is CD > BC ? equal to the square of the women in the room.
Once again, we get a range of possible values for
men
A B C D andwomen:m1w1,m4w2,m9w3,m16w4,m25w5,
and so on. A range of values and not a unique
(1) AD=20 (2) AB = CD information. We conclude that statement 2 alone
is also not sufficient. Our choices narrow to C or
Your Choice: A ( ) B ( ) C ( ) D ( ) E ( ) E. When we combine the two statements, we can
see that there is just one set of values for men
Statement 1 alone is not sufficient because we do
and women: w4 and w16 for a total of 20. The
not get a sense of the absolute values of CD and
combined information is sufficient to answer the
BC from the given information. Our choices are
question. We pick C as the answer.
B,C, or E at this stage. Statement 2 tells us that
3. If n is an integer, is (100-n) / n an integer?
AB and CD have equal values. That does not
mean much either. Our choices are C or E. When
(1) n > 4
we combine the two statements, we can generate
(2) n.n = 25
multiple scenarios: AB and CD could be both
equal to 1 and BC will then be equal to 18. Or,
AB and CD could be equal to 8, in which case Your Choice: A ( ) B ( ) C ( ) D ( ) E ( )
BC will be equal to 4, and so on. Since we get Whenever you see an algebraic expression or
BC > CD in one scenario and quite the opposite equation, try to rearrange things before you go to
examine the statements. In this problem, we can write
conclusion in the other scenario, we conclude
( 100 - n ) / n as 100/n - 1. If we get a sense from
that even the combined information does not statements 1 and 2 that n is a factor of 100, we will be
provide a unique value for the lengths of CD and able to answer the question posed.
BC. We have to pick E. Statement 1 tells us that n is greater than 4. Which means
that if n is 5, then it is a factor of 100. If n is 6, then it is
not. If n is 7, then it is not. If n is 10, then it is a factor of
100. And so on. Since we do not know what the true
value of n is, we get a range of possible YES/NO
scenarios. What do we conclude ? That statement 1 alone
is not sufficient to answer the question. Our choices are B,
C, or E.
Statement 2 tells us that n is equal to +5 or -5.
In either case, n is a factor of 100 and 100/n - 1 will be a
whole number. Statement 2 alone is sufficient and we pick
B.

21
©Educational Training Services, A Division of MLI Consulting, Inc.
GMAT DATA SUFFICIENCY TESTS 1-6 WITH EXPLANATIONS
THIS FILE IS MEANT FOR USE BY PARTICIPANTS REGISTERED IN OUR PREP COURSES
4. Last Friday a certain shop sold 3/4 of its 5. A jar containing 30 marbles, of which 20 are
inventory of sweaters. Each sweater sold for red and 10 are blue. If 9 of the marbles are
$20. What was the total revenue last Friday removed, how many of the marbles left in the jar
from the sale of these sweaters? are red?

(1) When the shop opened last Friday, there (1) Of the marbles removed, the ratio of the
were 160 sweaters in the inventory. number of red ones to the number of blue ones is
(2) All but 40 sweaters in the shop’s inventory 2:1.
were sold last Friday (2) Of the first 6 marbles removed, 4 are red.

Your Choice: A ( ) B ( ) C ( ) D ( ) E ( ) Your Choice: A ( ) B ( ) C ( ) D ( ) E ( )


To ace the data sufficiency section, you should We are looking for information pertaining to
be able to make the most sense of the given how many of the 9 marbles removed from the jar
information before you go to examine the were red in color.
statements and determine ahead of time what Statement 1 tells us that the number of red ones
information you are seeking. removed was 2x and the number of blue ones
In this problem, if N is the number of sweaters in was 1x . This information tells us that the
the inventory, we know that 3N/4 sweaters were number of red ones removed was 6 and that of
sold and N/4 were not. At $20 a sweater, the blue was 3. Good enough. We can determine
revenue from the sale of 3N/4 sweaters is how many red ones were left behind in the jar
$60N/4 = $15N. from this information alone. Therefore, our
choices at this juncture are A or D.
Statement 1 tells us that N = 160. Good enough. Statement 2 does not tell us much. We do not
We know that 3N/4 sweaters were sold and we know if the last 3 marbles removed were all red
can calculate the revenue. Statement 1 alone is or all blue or any combination of the two. Since
sufficient. Our choices are A or D. we are faced with a range of possible values for
Statement 2 tells us that the number of sweaters the removed red marbles, we have to conclude
not sold is equal to 40. Which means that N/4 = that statement 2 alone is not sufficient.
40 . We know that 3 times this number was We pick A as the answer.
sold. We can determine the quantity sold and
compute the revenue from the sales. Statement 2
alone is also sufficient to answer the question.
We pick D as the answer.

22
©Educational Training Services, A Division of MLI Consulting, Inc.
GMAT DATA SUFFICIENCY TESTS 1-6 WITH EXPLANATIONS
THIS FILE IS MEANT FOR USE BY PARTICIPANTS REGISTERED IN OUR PREP COURSES
6. Is the triangle ABC, whose angles are x,y and
z equilateral? 8. Is ax = 3 - bx ?

(1) x = y (1) x (a+b) = 3


(2) z=60 degrees (2) a = b = 1.5 and x = 1

Your Choice: A ( ) B ( ) C ( ) D ( ) E ( ) Your Choice: A ( ) B ( ) C ( ) D ( ) E ( )


Once again, we have an algebraic equation
An equilateral triangle has all sides of equal staring at us. Let us play around with the
length and all three angles equal to 60 degrees. equation and manipulate it into another form:
Do we get a sense of this from the statements ax = 3 - bx ? or ax + bx = 3 ?
given ? Or x (a + b) = 3 ?
Statement 1 tells us that two of the angles are Statement 1 tells us exactly what we are looking
equal. We know that ABC is at best an isosceles for. That x(a+b) = 3. Good enough to answer our
triangle. We have no other information to lead us question precisely. Statement 1 alone is
to conclude that ABC is equilateral or not. sufficient. Our choices are A or D.
Statement 1 alone is, therefore, not sufficient. Statement 2 gives us values for a, b, and x. If we
Our choices are B, C or E. plug these values into the expression x (a+b), we
Statement 2 tells us that one of the angles is 60 get 1 (1.5 + 1.5 ) = 3. Once again, the
degrees. The other two angles could have a information in statement 2 alone is also
range of possible values such that they add up to sufficient to answer the question.
120 degrees. Not good enough. Our choices Therefore, we pick D as the answer.
narrow to C or E.
When we combine the two statements, we can
conclude that if z=60 and x=y, then x and y must 9. What is the value of integer x?
also be equal to 60 degrees. Bingo. We pick C as
the answer. (1) x is a prime number
7. If w + z = 28, what is the value of wz? (2) 30 < x < 38
(1) w and z are positive integers
Your Choice: A ( ) B ( ) C ( ) D ( ) E ( )
(2) w and z are consecutive odd integers
Statement 1 tells us that x can have any prime
Your Choice: A ( ) B ( ) C ( ) D ( ) E ( ) number value and does not give us a unique
Statement 1 tells us that w and z can take on a value for x. Therefore, statement 1 alone is not
range of possible values such that they add up to sufficient to answer the question. Our choices
28. For instance, if w is 1, then z is 27; if w is 2, are B,C, or E.
then z is 26; if w is 10, then z is 18 and so on. Statement 2 tells us that x can have any value in
Not good enough for a unique value for the the range 31 through 37. Not precise enough.
variables. Our choices are B, C, or E at this Statement 2 alone is also not sufficient to answer
stage of the game. the question precisely. Our choices narrow to C
Statement 2 tells us that w and z are consecutive or E.
odd integers. Can we think of two consecutive When we combine the statements 1 and 2, we
odd numbers such that they add up to 28? We get two possible values for x : 31 and 37, both
can: 13 and 15. Since we are interested in the prime integers . Once again, we have more than
value for wz, it does not matter what value w one possible value from the given set of
takes and what value z takes. We know that if w statements. Not good enough.
is 13, then z is 15; if w is 15, then z is 13. In We pick E.
either case, the product wz will have a unique
value. Statement 2 alone is sufficient and
statement 1 alone is NOT sufficient. So we pick
B as the answer.

23
©Educational Training Services, A Division of MLI Consulting, Inc.
GMAT DATA SUFFICIENCY TESTS 1-6 WITH EXPLANATIONS
THIS FILE IS MEANT FOR USE BY PARTICIPANTS REGISTERED IN OUR PREP COURSES
10. While on a straight road, car X and Car Y are 12. What is the number of female employees in
traveling at different constant rates. If car X is Company X?
now 1 mile ahead of car Y. How many minutes
from now will car X be 2 miles ahead of car Y ? (1) If company X were to hire 14 more people
and all of these people were females, the ratio of
(1) Car X is traveling at 50 miles per hour and the number of male employees to the number of
car Y is traveling at 40 miles per hour. female employees would then be 16 to 9.
(2) 3 minutes ago car X was 1/2 mile ahead of (2) Company X has 105 more male employees
car Y. than female employees.

Your Choice: A ( ) B ( ) C ( ) D ( ) E ( ) Your Choice: A ( ) B ( ) C ( ) D ( ) E ( )


To determine the number of minutes it takes for Statement 1 tells us that ratio of Males (M) to
car X to get 1 more mile ahead of Car Y, we Females+14 (F+14) is 16/9. This does not mean
need the relative speed (difference of the two much because this ratio will be valid for values
speeds) of car X with respect to car Y. of M=32 and F = 4 or M=160 and F = 76. Since
we get a range of possible values, we have to
Statement 1 gives us the exact same information conclude that statement 1 alone is not sufficient.
we are seeking. Since the relative speed of Car Our choices are B, C, or E.
X is 10 miles with respect to Car Y, we know Statement 2 tells us that M = F + 105. Again,
that it will take another 6 minutes for Car X to this gives rise to a range of unlimited values for
gain a mile ahead of Car Y. Statement 1 alone is F and M. Not good enough. Our choices Narrow
sufficient. Our choices are A, or D. to C or E.
Statement 2 gives us information about the When we combine the two statements, we notice
relative speed in terms of time. Since the cars that we have two equations with two variables.
are traveling at constant rates, if car X was 1/2 We can solve the two equations to get the values
mile ahead of car Y, 3 minutes ago, then we for M and F. Good enough. We pick C.
know that it takes car X 6 minutes to gain an The equations are:
extra mile ahead of car Y. Statement 2 alone is M / (F+14) = 16/9
also sufficient to answer the question. Or 9M = 16F + 224 ............ (1)
We pick D. M = F + 105 ............(2)
11. In what year was Ellen born? 2 variables and 2 independent equations. Good
enough.
(1) Ellen’s brother Pete, who is 1 1/2 years older
than Ellen, was born in 1956.
(2) In 1975 Ellen turned 18 years old.

Your Choice: A ( ) B ( ) C ( ) D ( ) E ( )
Statement 1 tells us that if Pete was born during
January - June of 1956, then Ellen was born in
1957. If Pete was born during July-Dec. 1956,
then Ellen was born in 1958. Since we do not
have any indication of when Pete was born in
terms of month, we conclude that statement 1
alone is not sufficient to answer the question.
Our choices are B, C, or E.
Statement 2 tells us that Ellen must have been
born in 1957 so that she will turn 18 in 1975.
Good enough to answer the question. Statement
2 alone is sufficient and statement 1 alone is not
sufficient. We pick B.

24
©Educational Training Services, A Division of MLI Consulting, Inc.
GMAT DATA SUFFICIENCY TESTS 1-6 WITH EXPLANATIONS
THIS FILE IS MEANT FOR USE BY PARTICIPANTS REGISTERED IN OUR PREP COURSES
13. Is the integer x divisible by 36 ? 15. What is the value of a - b?

(1) x is divisible by 12 (1) a = b + 4


(2) x is divisible by 9. (2) (a - b) (a - b) = 16

Your Choice: A ( ) B ( ) C ( ) D ( ) E ( ) Your Choice: A ( ) B ( ) C ( ) D ( ) E ( )


Statement 1 tells us that x can have values that Statement 1 gives us the value for a - b as 4. A
are multiples of 12, values such as unique value and good enough information. Our
12,24,36,48,etc. If x is 12, 24, 48, 60 etc. Then choice is either A or D.
x is not divisible by 36. If x is 36,72, 108, and Statement 2 tells us that a-b is either +4 or -4 .
so on, then x is divisible by 36. Since we do not More than one possible value. Not good.
know the true value of x, we have to conclude We pick A.
that statement 1 alone is not sufficient. Our
choices are B, C, or E at this stage.
Statement 2 tells us that x is a multiple of 9. Not 16. Is rst = 1 ?
good enough. Because if x is 9, then it is not
divisible by 36. If it is 36, then it is . Therefore (1) rs = 1
statement 2 alone is also not sufficient. Our (2) st = 1
choices are C or E.
When we combine the two, we notice that the Your Choice: A ( ) B ( ) C ( ) D ( ) E ( )
least common multiple of 12 and 9 is 36 which Statement 1 does not give us any clue as to the
means that x must have values that are multiples value for t. T could have any value, including
of 36. Good enough to answer the question with fractional value. We cannot answer the question
the combined information. posed using information in statement 1 alone.
We pick C. Our choices are B, C , or E.
Statement 2 does not give us a value for r. Once
14. What is the average (arithmetic mean) of j again, r can have any number of values,
and k? including fractional. Statement 2 alone is also
not sufficient to answer the question.
(1) The average of j + 2 and k + 4 is 11 Our choice is either C or E.
(2) The average of j, k, and 14 is 10. When we combine the two information , we can
conclude only the following: That r, s, and t are
Your Choice: A ( ) B ( ) C ( ) D ( ) E ( ) each equal to 1, in which case rst will be equal
to 1 or that r and t are equal in value while s is
Statement 1 tells us that j+2+k+4 = 22. We can the reciprocal of the value of r or t. For
find out what the value for j+k / 2 will be. (8 . instance if r and t are equal to 1/3 and s is equal
You are not required to solve for the value in the to 3, then the conditions specified in statements 1
test. Do not waste time solving for the average and 2 will be met but the value for rst will be
value. It is enough for you to recognize that the 1/3.
equation is amenable to a unique solution) . Since we get a range of possible values from the
Therefore, statement 1 alone is sufficient to two information given, we have to conclude that
answer the question. Our choice is A or D. we cannot answer the question precisely even if
we combine the two statements. We have to pick
Statement 2 tells us that j+k+14 = 30. Once E.
again, we can compute the value for (j+k) / 2
from this equation. Statement 2 alone is also
sufficient to answer the question with a unique
value.
We pick D.

25
©Educational Training Services, A Division of MLI Consulting, Inc.
GMAT DATA SUFFICIENCY TESTS 1-6 WITH EXPLANATIONS
THIS FILE IS MEANT FOR USE BY PARTICIPANTS REGISTERED IN OUR PREP COURSES
17. In a certain office, 50 percent of the 18. Is xy < 6 ?
employees are college graduates and 60 percent
of the employees are over 40 years old. If 30 (1) x < 3 and y < 2
percent of those over forty have master’s (2) 1/2 < x < 2/3 and y.y < 64
degrees, how many of the employees over forty
have master’s degree? Your Choice: A ( ) B ( ) C ( ) D ( ) E ( )
Statement 1 tells that the values of x can be 2 or -10.
(1) Exactly 100 of the employees are college Similarly, the values of y can be 1.5 or -5.
graduates. If x is 2 and y is 1.5, then xy is 3, which is less than 6.
(2) Of the employees forty years old or less, 25 On the other hand, if x is -10 and y is -5, then xy is
percent have master’s degrees. +50, which is greater than 6. Since we do not know
the true values for x and y, we have to conclude that
statement 1 alone is not sufficient to answer the
Your Choice: A ( ) B ( ) C ( ) D ( ) E ( ) question. Our choices must be B,C, or E.
With word problems this long, you must reduce Statement 2 gives a range within which x values must
the information to equations you can work with. lie and also tells us that y is either +8 or -8. Since the
Let N be the number of employees in the greatest value that x can have is less than 2/3, with a
company. value of y = 8, we determine that the maximum
We have College Grads = 0.5N ...... (1) possible value for xy is 8.2/3 or 5 1/3. Which is less
40 + = 0.6N.........(2) than 6. Therefore, we conclude that the information in
statement 2 alone is sufficient to answer the question.
40+ & Master’s = 0.3. (0.6N) = 0.18N.... (3)
We pick B.
We are required to get a unique value for 0.18N.
19. Is x equal to 2?
If the statements give us numbers pertaining to
any of these categories, we can work with that.
(1) x, y and x + y are prime numbers
Let us examine statement 1.
(2) y is odd
Statement 1 tells us that the number of college
grads is equal to 100. Which means that
0.5N=100. We can determine the value for N Your Choice: A ( ) B ( ) C ( ) D ( ) E ( )
from this equation and plug the value in 0.18N to Statement 1 tells us that either x or y must be
get the information sought. Statement 1 alone is equal to 2. Why is this so? We know that the
sufficient to answer the question. Our choice is only even prime number is 2 and all the other
either A or D. prime numbers are odd numbers. If x+y is a
Statement 2 tells us that 25% of 0.4N = 0.1N prime number, then it must be odd. How do we
employees are under 40 years of age and have get an odd number? By adding an even number
master’s degree. This is information pertaining to an odd number. Therefore, when we add x and
to a new category and the information is in terms y, two prime numbers, we conclude that one of
of N. Not good enough to get a number value for them must be an even value so that the resulting
0.18N. Statement 2 alone is not sufficient number x+y will be odd.
We pick A. The only conclusion we can make from
statement 1 is that either x is 2 or y is 2. Not
good enough. Our choices are B, C, or E.
Statement 2 tells us that y is an odd number. We
have no information pertaining to x in statement
2. Statement 2, therefore, is not sufficient.
Our choice is either C or E.
When we combine the two statements, we reason
that either x or y is equal to 2, and if y is an odd
number, then x must be equal to 2. Bingo.
We pick C.

26
©Educational Training Services, A Division of MLI Consulting, Inc.
GMAT DATA SUFFICIENCY TESTS 1-6 WITH EXPLANATIONS
THIS FILE IS MEANT FOR USE BY PARTICIPANTS REGISTERED IN OUR PREP COURSES

20. In 1979 Mr. Jackson bought a total of n


shares of stock X and Mrs. Jackson bought a
total of 300 shares of stock X. If the couple held
all of their respective shares throughout 1980 ,
and Mr. Jackson’s 1980 dividends on his n
shares totaled $150, what was the total amount
of Mrs. Jackson’s 1980 dividends on her 300
shares ?

(1) In 1980 the annual dividend on each share of


stock X was $0.75
(2) In 1979, Mr. Jackson bought a total of 200
shares of stock X.

Your Choice: A ( ) B ( ) C ( ) D ( ) E ( )

Statement 1 tells us that each share received


$0.75 in dividends. We can determine how many
dollars Mrs. Jackson received on her 300 shares.
Statement 1 alone is sufficient to answer the
question. Our choice is either A or D.
Statement 2 tells that n = 200 which means that
if Mr. Jackson received $150 in dividend
income, then each share was worth $0.75 in
dividends. We can determine how many dollars
Mrs. J received on her holding of 300 shares.
Statement 2 alone is also sufficient to answer
the question.

We must choose option D.

27
©Educational Training Services, A Division of MLI Consulting, Inc.
GMAT DATA SUFFICIENCY TESTS 1-6 WITH EXPLANATIONS
THIS FILE IS MEANT FOR USE BY PARTICIPANTS REGISTERED IN OUR PREP COURSES
2. If p, q, x, y, and z are different positive
DATA SUFFICIENCY integers, which of the five integers is the
EXERCISE 2- Answer Keys median?
20 Questions -- 25 Minutes
CHOOSE: (1) p + x < q
A Statement (1) alone is sufficient but statement
(2) alone is not sufficient (2) y < z
B Statement (2) alone is sufficient but
statement (1) alone is not sufficient. A( ) B( ) C( ) D( ) E( )
C BOTH statement TOGETHER are To get the median value, we require information
sufficient, but NEITHER statement ALONE IS pertaining to the order of values for the integers p, q,
sufficient. x, y and z. Statement 1 tells us that q is bigger than
D EACH statement ALONE is sufficient both p and x. But we do not know whether p is
E Statements (1) and (2) TOGETHER are NOT greater than x or not. We also do not know about
sufficient. the other two values, y and z. Not good enough. Our
choices are B,C or E. Statement 2 tells us that y is
1. If the list price of a new car was $12,300, less than z. Not good enough because we do not
what was the cost of the car to the dealer? know about the other values. Our choices are C or E.
When we combine the two statements, we are not
(1) The cost to the dealer was equal to 80 percent any wiser because we do not know in what order we
of the list price should place x, p, y, and z. We must give up and
pick E as the answer.
(2) The car was sold for $11,070, which was
12.5 percent more than the cost to the dealer.
3. A certain employee is paid $6 per hour for an
8-hour workday. If the employee is paid 1 1/2
A( ) B( ) C( ) D( ) E( )
times this rate for time worked in excess of 8
Either statement alone is sufficient to answer hours during a single day, how many hours did
the question. Statement 1 tells us that cost is the employee work today?
80% of list price ( a known value). Good
enough. Our choices are A or D. Statement 2 (1) The employee was paid $18 more for hours
tells us that the selling price was 12.5% worked today than for hours worked yesterday.
higher than cost. Good enough to answer the
question precisely. We pick D. (2) Yesterday the employee worked 8 hours

A( ) B( ) C( ) D( ) E( )
Statement 1 alone is not sufficient to answer the question because
because $18.00 could mean 3 hours of regular time or 2 hours of extra
time. And we do not know how many hours the employee worked
“yesterday”. Our choices are B, C or E. Statement 2 tells us that the
employee worked 8 hours yesterday. Not good enough on its own.
We cannot answer the question: “How many hours did the employee
work today?” from this information. Choices are C or E.
When we combine the two statements, we know that the employee
worked 8 hours yesterday and that the employee worked 2 “overtime”
hours today for a total of 10 hours. We pick C.

28
©Educational Training Services, A Division of MLI Consulting, Inc.
GMAT DATA SUFFICIENCY TESTS 1-6 WITH EXPLANATIONS
THIS FILE IS MEANT FOR USE BY PARTICIPANTS REGISTERED IN OUR PREP COURSES
4. If n is a member of the set
(33, 36, 38, 39, 41, 42), 6. In the figure below, what is the length of AD?
What is the value of n ?
A_________B_________C_________D
(1) n is even
(1) AC = 6
(2) n is a multiple of 3
(2) BD = 6
A( ) B( ) C( ) D( ) E( )
A( ) B( ) C( ) D( ) E( )
Statement 1 tells us that n is even. We have 3
even values in the set: 36, 38, and 42. Not a Statement 1 alone is NOT sufficient because
unique solution. We conclude that statement 1 we do not get a value for AD from the given
alone is NOT sufficient. Our choices are B, C information. Choices are B, C or E at this
or E. Statement 2 tells us that n is a multiple of point. Statement 2 is not useful either because
3: We have 4 values: 33,36,39, and 42 for n. we still do not get a clue as to the value for
Not a unique solution either. Our choices AD. Choices narrow to C or E. When we
narrow to C or E. When we combine the two combine the two statements, we are no better
statements, we conclude that n must be either off because we do not know the relationship of
36 or 42. We do not get a unique value for n line segments AB, BC and CD to use the two
even with the combined information. We give statements to good use. We pick E.
up and pick E. 7. A retailer purchased a television set for x
percent less than its list price, and then sold it
5. What is the value of x ? for y percent less than its list price. What was
the list price of the television set?
(1) 2x + 1 = 0
(1) x = 15
(2) (x + 1 )² = x²
(2) x - y = 5
A( ) B( ) C( ) D( ) E( )
A( ) B( ) C( ) D( ) E( )

Statement 1 tells us the discount percentage


Statement 1 is good enough to get the value for
when the retailer bought the television. That is
x. (X = - ½ ) . Our choices are A or D.
not good enough to get the list price. Choice B,
Statement 2 must be manipulated to the form:
C or E.
x² + 2x + 1 = x² or 2x + 1 = 0 or x = - ½ .
Statement 2 gives us the difference in the
Good enough. Either statement alone is
discount rates. Once again, not sufficient to
sufficient to answer the question definitively.
answer the question. Even if we combine the two
We pick D.
statements, we are no better off. We need either
the purchase price or the selling price along with
discount %. We pick E.

29
©Educational Training Services, A Division of MLI Consulting, Inc.
GMAT DATA SUFFICIENCY TESTS 1-6 WITH EXPLANATIONS
THIS FILE IS MEANT FOR USE BY PARTICIPANTS REGISTERED IN OUR PREP COURSES
²
8. Is x greater than x ?
11.

B
²
(1) x is greater than 1
C
(2) x is greater than -1
A
A( ) B( ) C( ) D( ) E( )
The circular base of an above ground swimming
For x² to be greater than x, |x| must be pool lies in a level yard and just touches two
greater than 1. If 0 <x <1, then x² will be straight sides of a fence at points A and B, as
less than x. Statement 1 tells us that |x| is shown in the figure above. Point C is on the
greater than 1. Good enough. A or D. ground where the two sides of the fence meet.
Statement 2 tells us that x could be a How far from the center of the pool’s base is
negative fraction such as -1/4, in which point A ?
case x² is greater than x. If x is 1/4, then x
will be greater than x². If x is greater than (1) The base has area 250 square feet
1, x²will be greater than x. Too many
possibilities. Not a unique solution from (2) The center of the base is 20 feet from point
statement 2. We give up on statement 2 C.
and pick A.
A( ) B( ) C( ) D( ) E( )
9. What is the value of r/2 + s/2 ? In this problem, we are required to find the
value for the radius of the circular base from
(1) (r + s)/2 =5 the information given in statements 1 and 2.
Statement 1 tells us that the area of the
(2) r + s = 10 circular base is 250 sq. Ft. Or π . R² = 250.
Good enough. We can get the value for R.
A( ) B( ) C( ) D( ) E( ) Our choices are A or D. Statement 2 is not
much use because we get the following
We are required to get the average value for r picture:
and s. Statement 1 gives us that. Good B
enough. A or D. Statement 2 also lets us
calculate the average value for r and s. Either C 20’
statement alone is sufficient. We pick D.
A
We have the hypotenuse value in the above
10. If x, y, and z are numbers, is z = 18 ? right triangle BCO, but we need one other
information such as an anlge to determine the
(1) The average (arithmetic mean) of x, y, and z radius. We conclude that statement 2 alone is
is 6 not sufficient and pick A.

(2) x = -y

A( ) B( ) C( ) D( ) E( )
Statement 1 tells us that (x +y +z) ÷ 3 = 6 or x + y + z = 18 . Not much use,
because we cannot determine the value for z from this equation. We have a range of
possible values for z, and not a unique one. Our choices are B,C, or E. Statement 2
tells us that the absolute values of x and y are equal and that x and y have the opposite

30
©Educational Training Services, A Division of MLI Consulting, Inc.
GMAT DATA SUFFICIENCY TESTS 1-6 WITH EXPLANATIONS
THIS FILE IS MEANT FOR USE BY PARTICIPANTS REGISTERED IN OUR PREP COURSES
12. In 1979 Mr. Jackson bought a total of n
shares of stock X and Mrs. Jackson bought a
total of 300 shares of stock X. If the couple held
all of their respective stocks throughout 1980, 14. What is the value of (x/yz) ?
and Mr. Jackson’s 1980 dividends on his n
shares totaled $150, what was the total amount (1) x = y/2 and z = 2x/5
of Mrs. Jackson’s 1980 dividends on her 300
shares ? (2) x/z = 5/2 and 1/y = 1/10

(1) In 1980 the annual dividend on each share of A( ) B( ) C( ) D( ) E( )


stock X was $ 0.75
Statement 1 alone is not sufficient because
(2) In 1979 Mr. Jackson bought a total of 200 we get x/yz = 5/9x from the information
shares of stock X specified. B, C or E
Statement 2 tells us that x/yz = 5/20 or 1/4.
A( ) B( ) C( ) D( ) E( ) Good enough.
We pick B as the answer.
We need to know the value of the dividend to
answer the question. Statement 1 tells us
precisely that: That each dividend was worth
$0.75. Good enough. A or D. 15. An infinite sequence of positive integers is
Statement 2 tells us that Mr. J had 200 shares. called an “alpha sequence” if the number of even
The stem tells us that he received $150 in integers in the sequence is finite. If S is an
dividends. We can conclude that each share infinite sequence of positive integers, is S an
paid $0.75 in dividends, and answer the alpha sequence?
question posed. Either statement is sufficient
alone and we pick D. (1) The first ten integers in S are even.

13. If Sara’s age is exactly twice Bill’s age, what (2) An infinite number of integers in S are odd.
is Sara’s age?
A( ) B( ) C( ) D( ) E( )
(1) Four years ago, Sara’s age was exactly 3 To qualify as an alpha sequence, S must have a
times Bill’s age. finite number of even integers.
Statement 1 does not tell us how many integers
(2) Eight years from now, Sara’s age will be are even in the sequence. We simply know that
exactly 1.5 times Bill’s age. the first 10 are even. That is not sufficient.
B, C or E. Statement 2 tells us that S contains
A( ) B( ) C( ) D( ) E( ) an infinite number of odd integers. Not useful
The information in the stem is that Sara= 2. Bill. information. We require information about
Statement 1 tells us that S-4 = 3(B-4). even integers. C or E. Even when we combine
We have two equations and we can solve for Sara’s the two statements, we are clueless about how
age. A or D many even integers there are in S. We cannot,
Statement 2 tells us that S + 8 = 1.5(B+8) therefore, answer the question. We pick E.
Good enough in conjunction with the stem
information. Either statement alone is sufficient to
answer the question, and we pick D.

31
©Educational Training Services, A Division of MLI Consulting, Inc.
GMAT DATA SUFFICIENCY TESTS 1-6 WITH EXPLANATIONS
THIS FILE IS MEANT FOR USE BY PARTICIPANTS REGISTERED IN OUR PREP COURSES
16. If xy > 0, does (x-1)(y-1) = 1 ?
17. After winning 50 percent of the first 20
(1) x + y = xy games it played, Team A won all of the
remaining games. What was the total number of
(2) x = y games that Team A won ?

A( ) B( ) C( ) D( ) E( ) (1) Team A played 25 games altogether.

When we get an algebraic expression (2) Team A won 60 percent of all the games it
played.
such as this one, we will try to expand
it. A( ) B( ) C( ) D( ) E( )
(X-1)(Y-1) = XY-(X+Y) + 1.
Statement 1 tells us that X+Y = XY. We know that the team A won 10 of the 20
Which means that tbe above expression is equal to games it played. We need to know how
1. A or D many more it won, knowing that it won all of
Statement 2 tells us that x = y, which does not give the remaining games. Statement 1 tells us
a unique value for the expression. that the team played 25 games. Good enough.
(When you replace x with y, you get (y-1)2 and A or D. Statement 2 tells us that the team
you not know what value it has.) We pick A.
won 60% of all the games it played. If n were
the number of games, then the team won
0.6n which is equal to 10 + n - 20. We can
determine how many games the team A won
from both statements independently.
We pick D.

18. @ + $ = *

In the addition problem above, each of the


symbols @, $, and * represents a positive digit.
If @ < $, what is the value of $?

(1) * = 4

(2) @ = 1

A( ) B( ) C( ) D( ) E( )
Statement 1 tells us that @ + $ = 4.
Knowing that @ < $, we can conclude
that @ must be 1 and $ must be 3.
Good enough. A or D.
Statement 2 tells us that @ = 1. Not
good enough. $ can have a wide range
of values.
We conclude that statement 1 alone is,
but statement 2 alone is NOT, sufficient
and pick A.

32
©Educational Training Services, A Division of MLI Consulting, Inc.
GMAT DATA SUFFICIENCY TESTS 1-6 WITH EXPLANATIONS
THIS FILE IS MEANT FOR USE BY PARTICIPANTS REGISTERED IN OUR PREP COURSES
19. Cancellation Fees Schedule
20. Is 5K less than 1,000?
Days prior to Percent of
Departure Package Price (1) 5K+1 is greater than 3,000

46 or more 10% (2) 5K-1 is 500 less than 5 to the power of k.


45 - 31 35% A( ) B( ) C( ) D( ) E( )
30 - 16 50%
15 - 5 65% We need to know whether k is less than 5
4 or fewer 100% because 55 = 3125 and 54 = 625.

The table above shows the cancellation fee Statement 1 tells us that
schedule that a travel agency uses to determine 5k+1 > 3000
the fee charged to a tourist who cancels a trip Or 5. 5k > 3000 or 5k > 600
prior to departure. If a tourist canceled a trip The fact that 5k > 600 does not tell us
with a package price of $1,700 and a departure definitively that 5k < 1000 . Greater than
date of September 4, on what day was the trip 600 could mean 900 or 5000 Statement 1 is
canceled? not good enough for a unique determination
of the value for 5k. We must, therefore,
(1) The cancellation fee was $595 move on to examine statement 2. Our
choices are B, C or E at this point in time.
(2) If the trip had been canceled one day later,
the cancellation fee would have been $255 more. Statement 2 tells us that
5k-1 = 5k – 500
A( ) B( ) C( ) D( ) E( ) or 5-1.5k = 5k – 500
or 1/5 . 5k = 5k – 500
From statement 1, we know that the cancellation Let us multiply both sides by 5 to get rid of
amount was 35% of the tour charge. Which 1/5 on the left. We get
means that cancellation took place during 30-45 5k = 51.5k – 2500
days. That is a broad range of days and not Let us group the 5k terms together to get:
precise enough to answer the question. B, C, or 4. 5k = 2500 or 5k = 625 = 54
E. Statement 2 tells us that if the cancellation had We conclude that 5k < 1000 because we
taken place one day later, then the additional fee know that 5k = 625. Statement 2 alone is
would be 15% of the tour fee. Which means that sufficient but statement 1 alone is not
the cancellation occurred on the 31st or the 16th sufficient to answer the question
day. Not precise enough. C or E. When we definitively.
combine the two statements, we can conclude We must pick B as the answer.
that the cancellation must have occurred on day
31. We pick C.

33
©Educational Training Services, A Division of MLI Consulting, Inc.
GMAT DATA SUFFICIENCY TESTS 1-6 WITH EXPLANATIONS
THIS FILE IS MEANT FOR USE BY PARTICIPANTS REGISTERED IN OUR PREP COURSES

DATA SUFFICIENCY 2. If n is an integer, is n + 2 a prime number ?


EXERCISE 3-Answer Keys
20 Questions -- 25 Minutes (1) n is a prime number
CHOOSE:
A Statement (1) alone is sufficient (to give a (2) 30 < n < 40
unique value for the information sought) but
statement (2) alone is not sufficient A ( ) B( )C(X) D( ) E ( )
B Statement (2) alone is sufficient but
statement (1) alone is not sufficient. We need some additional information about n in
C BOTH statement TOGETHER are order to determine whether n+2 is a prime
sufficient, but NEITHER statement ALONE IS number or not.
sufficient. Statement 1 tells us that n is a prime integer.
D EACH statement ALONE is sufficient How many prime integers can we think of? Lots
E Statements (1) and (2) TOGETHER are NOT and lots. Is this a unique solution? Hardly.
sufficient. Statement 1 is not sufficient by itself. B,C, or E.
Statement 2 tells us that n lies in the range
1. The results of a certain experiment included 6 between 30.0001 and 39.999, inclusive. Once
data values that were all multiples of the same again, a range of possible values for n. Not good
number c, namely, c, 8c, 2c, 5c, 4c, and 4c. Was for a unique solution. C or E.
the average (arithmetic mean) of the 6 data When we combine the two statements, we notice
values greater than 8? that n is a prime integer and lies in the range :
(1) c < 4 30 < n < 40. How many prime numbers can we
(2) c > 2 find in this range? Two: 31 and 37. We notice
that n can be either of these values. But the
question is not about n, but about n+2. For either
A ( ) B(x)C( ) D( ) E ( ) of these values for n: 31 or 37, (n+2) is not a
prime number. (Because 33 and 39 are not prime
Let us compute the average of the six values by numbers.)
adding the values and dividing the sum by the Therefore, we conclude that we can answer the
number of values. question definitively by combining the two
The average in terms of c = statements. Notice how we get 2 possible values
1/6 (c+8c+2c+5c+4c+4c) = 24c/6 = 4c for n, but a unique solution for (n+2) in terms of
The question is: Is 4c definitely more than 8 or a definite answer confirming that n+2 is NOT a
definitely NOT more than 8? Or, paraphrasing prime integer. We must pick choice C.
the question, is c definitely more than 2 or
definitely not greater than 2?
Statement 1 alone is not sufficient because if c is
less than 4, it could be 3, 2 ½ , 2, 1 ½ and so on.
In some scenarios, c is greater than 2, and in
others, it is not. We are not logically certain
whether c is definitely greater than 2 or not.

Statement 2 alone is sufficient to answer the


question. If c is greater than 2, according to
statement 2, then that ought to definitively
answer the question is 4c > 8.
We must choose option B.

Try to simplify the stem question so that you can


have a better handle on the statements presented
to you.

34
©Educational Training Services, A Division of MLI Consulting, Inc.
GMAT DATA SUFFICIENCY TESTS 1-6 WITH EXPLANATIONS
THIS FILE IS MEANT FOR USE BY PARTICIPANTS REGISTERED IN OUR PREP COURSES
3. If t is not equal to zero, is r a positive 4. How many minutes does it take for a circular
number? shaped illumination to quadruple its area?

(1) rt = 12 (1) The initial diameter of the circular shaped


illumination is 10 in.
(2) r + t = 7
(2) The circumference of the circular shaped
A ( ) B( )C(X) D( ) E ( ) illumination increases at the rate of pi. In/minute

We learn that t is not equal to 0, but it could be A ( ) B( )C( ) D( ) E ( )


positive or negative, integer or “not-an-integer,”
We need to get a handle on r. When will the area of a circular shaped object
Statement 1 tells us that rt = 12. Is this good quadruple? When the radius doubles, because the area
enough to let us know what r is about? Hardly. R is a function of (radius)2. To answer the question, we
and t could be both negative or both positive, and need to know the initial value of the radius, and the rate
at which the radius is changing.
we cannot answer the question definitively. Our Statement 1 tells us what the radius is, but does not tell
choices narrow to B, C, or E. us at what rate the radius is increasing. We cannot
Statement 2 tells us that r+t = 7. Is this good answer the question using statement 1 alone. Our
enough information to conclude that r is positive? choices are B, C or E.
No. Why? Because r could be -2 and t could be Statement 2 tells us at what rate the radius is increasing.
+9 or r could be +11/3 and t could be +52/3. We If the circumference is increasing at the rate of pi inches/
get solutions that are “all over the map”. Our minute, then we know that the radius is increasing at the
choices are C or E. rate of 1/2 inch/min. We get one half the information we
When we combine the two statements, we notice are seeking. Our choices are C or E.
We can answer the question by combining the two
that we have two variables, and two independent
statements, because we get the radius information from
equations. We can determine what r is all about. 1, and the rate of change from 2. We must pick C.
C is the answer. (In fact, the values for r and t are 4
and 3 or 3 & 4, both positive).

4. If x is an integer, is y an integer ?

(1) The average (arithmetic mean) of x, y, and


y - 2 is x .

(2) y - x = 1

A ( ) B( )C( ) D( X) E ( )

Statement 1 tells us that (x+y+y-2) = x


3
Or x + 2y - 2 = 3x or 2(y-1) = 2x or y-1 = x What do we conclude from this? That x and y are
consecutive integers. If x is an integer, then y MUST be an integer.
Our choices are A or D.
Statement 2 tells us that y - x = 1 or y = x+1
Once again, what this means is that x and y are consecutive integers. If x is an integer, so is y.
Statements 1 and 2 independently are sufficient to answer the question. We must pick D.
Notice how we set up an equation using the information in statement 1 before concluding that the
information is useful for a unique answer. Be sure to express in mathematical terms any information
in verbal form, before you conclude one way or the other.

35
©Educational Training Services, A Division of MLI Consulting, Inc.
GMAT DATA SUFFICIENCY TESTS 1-6 WITH EXPLANATIONS
THIS FILE IS MEANT FOR USE BY PARTICIPANTS REGISTERED IN OUR PREP COURSES
5. The inside of a rectangular carton is 48
centimeters long, 32 centimeters wide, and 15
centimeters high. The carton is filled to capacity
with k identical cylindrical cans of fruit that
stand upright in rows and columns as shown We know the dimensions of the box, and if we
below. If the cans are 15 centimeters high, what know the radius of the cans, we can answer the
is the value of k ? question.
Statement 1 tells us that each can has a radius
of 4 cm. We can determine how many cans of
8 inches diameter can be placed along the
length (48 cm), and width (32 cm). Good
enough. Our choices are A or D.
(1) Each of the cans has a radius of 4 Statement 2 tells us that 6 cans can be placed
centimeters. along the length 48 cm. This means that the
diameter of each can is 8 cm. We can
(2) 6 of the cans fit along the length of the determine how many we can place along the
carton. width (32 cm), and answer the question
definitively. Either statement alone is
A ( ) B( )C( ) D( X) E ( ) sufficient to answer the question. D is it.

36
©Educational Training Services, A Division of MLI Consulting, Inc.
GMAT DATA SUFFICIENCY TESTS 1-6 WITH EXPLANATIONS
THIS FILE IS MEANT FOR USE BY PARTICIPANTS REGISTERED IN OUR PREP COURSES
6. If ™ is a positive integer, is the value of b - a
at least twice the value of (1) When a is divided by 8, it leaves a
remainder of 3.
3k - 2k ?
(1) a is equal to 2 to the power of ™+1 and b is (2) When b is divided by 8, it leaves a remainder
equal to 3 to the power of ™+1 of 5.

(2) ™ = 3 A ( ) B( )C(X) D( ) E ( )
Statement 1 tells us that a = 8.p + 3, where p
A (X) B( )C( ) D( ) E ( ) represents all the other factors of (a-3). Why
do we write the information in this manner?
We need to determine whether b-a μ (3k - 2k) Let us say that a = 19. We can write 19=8.2 + 3
Statement 1 tells us that a = 2k+1 and b = 3k+1 We notice that we do not have any information
So. What is b - a ? b - a = 3k+1 - 2k+1 about b in this statement. Our choices are B, C or E.
We can write b - a = 3. 3k - 2.2k Statement 2 tells us that b = 8.q + 5. Is this
Or b - a = 2 { 1.5 (3k - 2k)} statement sufficient to answer the question about
(a+b)? We don’t think so. C or E.
What did we do here? Simply factored the
When we combine the two statements we can write
expression, so that we can determine if (b-a) is that (a+b) = 8(p+q) + 8. Is (a+b) divisible by 8?
twice something or not. What do we see here? You bet. Why is that? Because (a+b) = p+q+1
That (b-a) is indeed more than twice 3k - 2k. 8
Our choices are A or D. p+q+1 is a whole number and we must conclude that
Statement 2 gives us a value for k, but does not 8 is a factor of (a+b).
tell us what a and b are. We cannot determine If the problem was specified in such a manner that
what (b-a) will be in the absence of any we got an expression for a+b = 8(p+q) + 7, we can
information about a and b. We conclude that still answer the question in the negative: That (a+b)
is not divisible by 8. As long as the information is
statement2 alone is not sufficient, but statement 1
good for a definite answer, the information alone or
alone is sufficient. We must pick A. combined is good enough.
We pick C in this problem.
7. If P and Q are each circular regions, what is
the radius of the larger of these regions?

(1) The area of P plus the area of Q is equal to


90 pi.

(2) The larger circular region has a radius that is


3 times that of the smaller circular region.

A ( ) B( )C(X) D( ) E ( )
Statement 1 tells us that pi.rp2 + pi.rq2 = 90.pi
Or rp2 + rq2 = 90
Is this good enough information to answer the question? Nope. We need to know the
values for these radii.
We conclude that statement 1 alone is not sufficient.
Our choices narrow to B, C or E.
Statement 2 tells us that one of the radius is 3 times the other value. If rp is the larger of the
two, then
rp = 3.rq (it does not matter which is the larger of the two radii) Is this good enough? No.
We need to know the number value for the radii. Our choices are C or E.
h bi h h i d d i d

8. Is a+ b divisible by 8 ?

37
©Educational Training Services, A Division of MLI Consulting, Inc.
GMAT DATA SUFFICIENCY TESTS 1-6 WITH EXPLANATIONS
THIS FILE IS MEANT FOR USE BY PARTICIPANTS REGISTERED IN OUR PREP COURSES
9. What is the ratio of x : y : z ?
11. On a Friday Morning, a certain machine ran
(1) z = 1 and xy = 32 continuously at a uniform rate to till a
production order. At what time did it completely
(2) x = 2y and y = 4z fill the order that morning ?

A ( ) B(X)C( ) D( ) E ( ) (1) The machine began filling the order at 9:30


a.m.
Statement 1 is not sufficient because we do not
know what x and y are. If xy = 32, then x and (2) The machine had filled 1/2 of the order by
y could be 1 and 32, or 2 and 16, or 4 and 8, or 10:30 a.m. and 5/6 of the order by 11:10 a.m.
8 and 4, or 16 and 2, or 32 and 1. Not a unique
solution here. Our choices are B, C or E. A ( ) B( )C( ) D( ) E ( )
Statement 2 tell us exactly what we need to The question is: When did the machine FINISH
know: A relationship between x , y and z. filling the order?
If x = 2y and y = 4z, then we can write Statement 1 tells us the start time, but does not tell
x = 2y = 8z. us at what rate the machine is working. We cannot
We know that x:y:z must be 8:4:1. determine at what time the machine completed
Statement 2 alone is sufficient to answer the work. Statement 1 is useless, and our choices are
question, and we must pick B as the answer. B, C or E.
Statement 2 tells us that the machine, working at a
constant rate, completed (5/6 - 1/2) = 1/3 of the
10. If 3m = 5n , what is the value of m + n ? order in 40 minutes. We know that the machine
had done 5/6 of the order at 11:10 am, and needs
(1) 2m + n = 26 to finish 1/6 of the order. If it does 1/3 of the order
in 40 minutes, the machine will take 20minutes
(2) If the value of m were increased by 2, then more to do 1/6 of the order. We can conclude that
it will be twice the value of n. the machine will complete the order at 11:30 am.
Statement 2 alone is sufficient, but 1 alone is not.
A ( ) B( )C( ) D(X ) E ( ) We must pick B.

We have some information specified in terms of m


and n, along with the question. We must use this
information in conjunction with information in
statements 1 and 2 to determine whether we can
answer the question or not.
Statement 1 tells us that 2m+ n = 26. When we
combine this with 3m = 5n, we have unique
solution for m and n staring us in the face. A or D
Statement 2 tells us that m+2 = 2n. Again, when
we combine this information with 3m = 5n, we can
determine the values for m and n, and answer the
question definitively. D is the answer.

38
©Educational Training Services, A Division of MLI Consulting, Inc.
GMAT DATA SUFFICIENCY TESTS 1-6 WITH EXPLANATIONS
THIS FILE IS MEANT FOR USE BY PARTICIPANTS REGISTERED IN OUR PREP COURSES
12. A ladder is propped up against a vertical 13. Can the positive integer n be written as the
wall such that the bottom of the ladder makes an sum of two different positive prime numbers ?
angle of 60 degrees to the ground. The ladder
slips down and the bottom of the ladder moves (1) n is greater than 3
away from the wall. In this position, the bottom
of the ladder makes an angle of 45 degrees to the (2) n is odd.
ground. How much farther did the ladder move
from the wall when it slipped? A ( ) B( )C( ) D( ) E (X)
Statement 1 tells us that n > 3.
(1) The ladder is 10 meters long If n is 7, then we can write 7 = 2 + 5, the sum of
two different prime integers.
(2) The bottom of the ladder was 5 meters from If n is 11, then we cannot write 11 as the sum of
the wall in its original position. two different positive prime numbers.
Since we do not know what specific value n has,
A ( ) B( )C( ) D( X) E ( ) we cannot answer the question definitively from
This is the picture emerging from the information in the statement 1 information. B, C or E.
the problem: Statement 2 tells us that n is odd. We have seen
We need to know the value for “x” in the diagram. that for values 7, and 11, we get two different
scenarios: If n is 7, then we can write the value
as the sum of two primes; If n is 11, then we
cannot. Statement 2 also gives us “all-over-the-
map” solution. Not good enough. C or E.
When we combine the two statements, we notice
45o 60o that we have covered all bases with 7 and 11,
x 5m1 both values greater than 3, and get conflicting
We also notice that the triangles are 60-30-90 before answers.
the ladder slipped, and 45-45-90 after the ladder Even the combined information is not good for a
slipped. If we know the value for any one side, we unique answer. We must pick E.
can determine all the other side values because the
sides of a 45-45-90 triangle are in proportion 1:1:\/2
and those of 30-60-90 are in proportion 1:\/3:2.
Statement 1 tells us that the bottom of the ladder is
5m from the wall. We know that the length of the
ladder is 10 feet, and in the new position (45-45-90),
the bottom of the ladder must be 10/\/2 = 5\/2 meters
from the vertical wall. The slippage, x = 5\/2 - 5 =
5(\/2 -1) = 2 feet(approx). Statement 1 alone is
sufficient. Our choices are A or D.
Statement2 tells us that the length of the ladder is 10
feet. We can use this information along with the
properties of 30-60-90 and 45-45-90 triangles to
determine the value for x in the diagram.
Statement 2 is also independently sufficient to
answer the question.
We must pick D as the answer.

39
©Educational Training Services, A Division of MLI Consulting, Inc.
GMAT DATA SUFFICIENCY TESTS 1-6 WITH EXPLANATIONS
THIS FILE IS MEANT FOR USE BY PARTICIPANTS REGISTERED IN OUR PREP COURSES
14. What is the last number in a set comprising
six consecutive odd integers ?
15. Town T has 20,000 residents, 60 percent of
(1) The average (arithmetic mean) of the six whom are female. What percent of the residents
consecutive odd integers is 36. were born in Town T ?

(2) Twice the average (arithmetic mean) of the (1) The number of female residents who were
six consecutive odd integers is equal to the sum born in Town T is twice the number of male
of the first and the last number in the set. residents who were not born in Town T.

A ( ) B( )C( ) D( ) E ( ) (2) The number of female residents who were


not born in Town T is twice the number of
How do we represent 6 consecutive odd integers? female residents who were born in Town T.
2n +1, 2n+3, 2n+5, 2n+7, 2n+9, 2n+11
What is the sum of these integers? 12n + 36. A ( ) B( )C(X) D( ) E ( )
What is the average of these integers?12n+36 This is a problem best dealt with using a matrix
6 that will look like this:
Or the average = 2n + 6.
Statement 1 tells us that the averageis 36. MEN WOMEN TOTAL
The equation for this information is: BORN 6000
c 2N1 X2 10000
c
2n + 6 = 36 or n = 15. 4000
2
We can determine that the last number in the NOT N1 2X2 10000
c
set must be 41. Statement 1 alone is sufficient. BORN 2000 8000
c 2
Our choices are A or D.
Statement 2 is not so precise. TOTAL 8000 12,000 20,000

Twice the average = First integer + last integer


Or 2 ( 2n+ 6) = 2n+ 1 + 2n+ 11 We need to know how many men and women were
Or 4n + 12 = 4n + 12. born in Town T.
Is this information leading us anywhere? Statement 1 tells us that Men/Not Born is 1/2 the
Up the garden path, for sure. number for Women/Born. The information is shown
What do we make of statement 2? in the matrix with the suffix 1. But this information
Not good enough to answer the question. is not sufficient to answer the question.
We have determined that statement 1 alone is Statement 2 tells us that number of women born in
town T is 1/2 the number not born there. This
sufficient, but statement 2 alone is not.
information helps us determine that the number of
What choice do we pick under the women born in Town T is 1/3 of 12,000 = 4000;
circumstances? number not born in Town T is 2/3 the total number
Choice A as in “amen.” for women = 2/3 or 12,000 = 8000. But this
information alone does not tell us how many men
were born in Town T.
When we combine the two statements, we can
determine that the number of men born in Town T is
6000, and the total number of men and women born
in town T is 10000. Choice C.

40
©Educational Training Services, A Division of MLI Consulting, Inc.
GMAT DATA SUFFICIENCY TESTS 1-6 WITH EXPLANATIONS
THIS FILE IS MEANT FOR USE BY PARTICIPANTS REGISTERED IN OUR PREP COURSES
16. A right triangular region PQR lies in a
rectangular coordinate plane such that each of its 17. If both x and y are non-zero numbers, what
sides, PQ and PR, lies parallel to the rectangular is the value of y/x ?
coordinate axes. Is the right triangle isosceles ?
(1) x = 8
(1) One of the points Q on the hypotenuse RQ
has the coordinates (3, 4).
(2) y³ = x²
(2) One of the points R on the hypotenuse RQ
has the coordinates (-2, -1)
A ( ) B( )C(X) D( ) E ( )
A ( ) B( )C(X) D( ) E ( ) We need to know the values for x and y to
answer the question.
Statement 1 alone is not sufficient because we
do not know what y is.
Q(3,4)1 Our choices are B, C or E.
Statement 2 alone is also not sufficient because
we cannot establish a specific set of values for
x and y from the given information. Our
choices narrow to C or E.
(-2,-1)2 R P(3,-1)c When we combine the two statements, we can
determine that y must be 4, and x is 8, and the
Statement 1 gives us the coordinates for Q. ratio is 4/8 = 1/2. Answer C.
That is not sufficient information because we (How do we know that y is 4? Because y3= x2
need to know the values for R to be able to and if x = 8, then 3
y = 82 = 64 or y = 4)
answer the question. Our choices are B, C or
E.
Statement 2 gives us the coordinates of R.
We do not have any information about Q.
Our choices narrow to C or E.
When we combine the two statements, we
can determine that P must be (3, -1) and that
PQ must be 5, and PR must be 5 as well. We
can conclude that the triangle is isosceles
right triangle using the combined
information. We must pick choice C.

41
©Educational Training Services, A Division of MLI Consulting, Inc.
GMAT DATA SUFFICIENCY TESTS 1-6 WITH EXPLANATIONS
THIS FILE IS MEANT FOR USE BY PARTICIPANTS REGISTERED IN OUR PREP COURSES
18. If x = 0.rstu where r, s, t, and u each
represent a non-zero digit of x, what is the value 19. If x and y are integers between 10 and 99,
of x ? inclusive, is (x - y) /9 an integer ?

(1) r = 3s = 2t = 6u (1) x and y have the same two digits but in the
reverse order.
(2) The product of r and u is equal to the
product of s and t (2) The tens’ digit of x is 2 more than the units’
digit and the tens’ digit of y is 2 less than the
A ( ) B( )C( ) D( ) E ( ) units’ digit.
X is defined as a decimal number, and the digits A ( ) B( )C( ) D( ) E ( )
r,s,t, and u represent the tenth, hundredth,
thousandth, and ten-thousandth digits. Can we
determine what the values for these digits must be
Statement 1 tells us that x and y have the same
using the information in statements 1 and 2 digits but in the reverse order. You must know
independently or combined? Let us see. that if you take the difference of any two digit
Statement 1 tells us that there is a relationship integers whose digits are the same but in the
between the digits. Because we know that a digit is reverse order, the difference is always divisible
a positive integer having a value between 0 and 9, by 9. (See module II for explanation of how
and because r,s,t, and u are defined as non-zero this is so) For example, the numbers could be
digits, they must have values that lie between 1 and 13 and 31, 24 and 42, or 56 and 65. As long as
9. The value for u is constrained by the relationship the two numbers have the same two digits but
r = 6u. We conclude that u must be 1, because if u is
in the reverse order, the difference is always
2, then r must be 12, and r cannot be a digit with a
value of 12. If u=1, then r = 6, s = 2, and t = 3. (We divisible by 9.
have used the relationship specified to determine the Our choices are A or D.
values for r,s t, and u.). Now that we know what Statement 2 tells us x and y could be any of the
r,s,t, and u are, we know that x = 0.6231 following values:
Choices are limited to A or D. x: 20, 31,42,53,64,75,86,97
Statement 2 is not good for a unique solution. ru=st y: 13, 24, 35, 46, 57, 68, 79
is valid for a range of values for r,s,t, and u. If x is 20 and y is 13, then the difference is not
We must pick choice A. divisible by 9. But if x is 42 and y were 24,
then the difference is. We are getting “all-over-
the-map” solution with this information. We
must conclude that statement 2 is not good for
a unique solution.
However, statement 1 alone is good for a
definite answer.
We must pick choice A.

42
©Educational Training Services, A Division of MLI Consulting, Inc.
GMAT DATA SUFFICIENCY TESTS 1-6 WITH EXPLANATIONS
THIS FILE IS MEANT FOR USE BY PARTICIPANTS REGISTERED IN OUR PREP COURSES

20. Is the positive integer n equal to the square


an integer ?

(1) For every prime number p, if p is a divisor


or n, then so is p-squared..

(2) Square root of n is an integer.

A ( ) B( )C( ) D( ) E ( )

The question is: can we write n = k2 where k


is any integer?
Statement 1 tells us that p2 is a factor of n.
We know that p2 is one factor, and there are
other factors which we will collectively
represent by the letter r.
We get n = r. P2
Since we do not know whether we can write r
as a square of an integer, we cannot
determine from statement 1 alone whether n
can be expressed as a perfect square. Our
choices are B, C or E.
Statement 2 tells us that \/n = integer
When we square both sides, we get
n = (integer)2
Voila! We got the answer we are looking for.
Statement 2 alone is sufficient to answer the
question definitively, but statement 1 alone is
not. The corresponding choice is B.

43
©Educational Training Services, A Division of MLI Consulting, Inc.
GMAT DATA SUFFICIENCY TESTS 1-6 WITH EXPLANATIONS
THIS FILE IS MEANT FOR USE BY PARTICIPANTS REGISTERED IN OUR PREP COURSES

DATA SUFFICIENCY 4
Time : 25 Minutes 20 Questions.

1. Is x<y?
Directions: Each of the Data Sufficiency
problems below consists of a question and two
(1) x2 < y2
statements, labeled (1) and (2), in which certain
data are given. You have to decide whether the
(2) x < y + 1
data given in the statements are sufficient for
answering the question. Using the data given in Statement 1 tells us that x < y or x > -y. We
the statements plus your knowledge of cannot conclude from this statement that x < y.
mathematics and everyday facts (such as the In real life terms, x could be -2 and y could be
number of days in July or the meaning of 3, in which case x < y. Or x could be +3 and y
counterclockwise), you are to fill in oval, could be -8, in which case x2 < y2 , but x > y.
We are getting “all-over-the-map” solution
A if statement (1) ALONE is sufficient, but with this statement. Our choices are B, C or E.
statement (2) alone is not sufficient to answer the Statement 2 tells us that x < y+1. This
question asked; information is not sufficient to conclude that x
< y because x could be y + 0.2 or x could be y
B if statement (2) ALONE is sufficient, but - 5. Both values for x are consistent with the
statement (1) alone is not sufficient to answer the specification x < y+1. Our choices are C or E.
question asked; When we combine the two statements, we
conclude that x must lie in the range:
C if BOTH statements TOGETHER are -y < x < y
sufficient to answer the question asked, but For some values of x in this range, x will be
NEITHER statement alone is sufficient. greater than y; for others, x will be less than y.
We cannot conclude on any basis that x < y
D if EACH statement ALONE is sufficient to even when we combine the two statements.
answer the question asked; We must pick E. On a number line, we will
represent this information as follows:
E if statement (10 and (2) TOGETHER are NOT
sufficient to answer the question asked, and -y 0 +y
additional data specific to the problem are X
needed. If x is to the left of 0 ( -Y < X < 0), then x > y.
If x is to the right of 0 (0 < X < Y), then x < y.
All numbers used are real numbers. A figure in Can you see the possibilities here?
the data sufficiency problem will conform to the E is the best answer.
information given in the question, but will not
necessarily conform to the additional
information given in statements (1) and (2).

All figures lie in a plane unless otherwise


indicated.

In questions that ask for the value of a quantity,


the data given in the statements are sufficient
only when it is possible to determine exactly one
numerical value for the quantity.

44
©Educational Training Services, A Division of MLI Consulting, Inc.
Not to be reproduced without our express written consent
Name of the Registrant: PARTICIPANTS Registration Number: 0098TURBO-2452USA
2. Machine A runs at a constant rate and
produces a lot consisting of 100 bolts in 30 4. Is triangle ABC equilateral?
minutes. How much less time would it take to
produce the lot of bolts if both machines A and (1) Two of the angles are 60 degrees each.
B were run simultaneously?
(2) One side measures 6 inches.
(1) Both machines A and B produce the same
number of bolts per hour. We need to know if the angles are 60 degrees
each or if the sides are the same length.
(2) It takes machine A twice as long as it takes We need to use our knowledge of the sum of
machines A and B, running simultaneously, to the internal angles of a triangle. We know
produce the same lot of bolts. that the internal angles add up to 180o.
Statement 1 tells us that two angles are 60o
We know the rate at which machine A is each. We can conclude that the third angle
running. We need to know the rate at which must be 60o as well. The triangle must be
machine B is running so that we can determine equilateral. Our choices are A or D.
how long it will take for both machines to Statement 2 tells us that one side is 6 inches.
complete the job working together at their We have no information about the other two
respective constant rates. sides. We cannot conclude on any basis that
Statement 1 tells us that Machine B runs at the the triangle is an equilateral one using
same rate as machine A. We can conclude that statement 2 alone.
both machines running together can finish the We conclude that statement 1 alone is, but
job in 15 minutes or in 15 fewer minutes. not statement 2 alone is, sufficient. We pick
Statement 1 alone is sufficient. Our choices are A.
A or D.
Statement 2 tells us that Machine A takes twice
as long as Machines A and B together. This
information can mean only one thing: That
machine B operates at the same rate as Machine
A. We can live and work with this information
also. We must pick D.
3. Is p a prime number?

(1) p+1 is a prime number.

(2) p is even integer.

Statement 1 tells us that p+1 is a prime number. If p+1 is 3, then p is 2, a prime number.
But if p+1 is 5, then p is 4, not a prime number. Can you see that we are beginning to get
conflicting results with just two values tested for p+1? What must we conclude? That
statement 1 alone is not sufficient. Our choices are B, C or E.
Statement 2 tells us that p is even integer.
If p is 2, then p+1 is a prime integer, and so is p. If p is any other even integer, then it
cannot be a prime integer, because the only even value that is prime integer is 2. Since
there is no basis for us to conclude that p is equal to 2, we must conclude that statement 2 is
also not sufficient. Choices narrow to C or E.
When we combine the two statements, we notice that we are dealing with the same
uncertainties encountered under the two statements independently. We must go with E.

45
©Educational Training Services, A Division of MLI Consulting, Inc.
Not to be reproduced without our express written consent
Name of the Registrant: PARTICIPANTS Registration Number: 0098TURBO-2452USA
5. If n is an integer, is (100 - n) / 2n 7. Buckets X and Y contained only water and
an integer? bucket Y was half full. If all the water in bucket
X was poured into bucket Y, what fraction of the
(1) n < 60 capacity of Y was then filled with water?
(1) Before the water from X was poured into Y,
(2) n is divisible by 10. X was 1/4 full.
Statement 1 tells us that n < 60. If n = 10,
then the expression being tested is not an (2) X has twice the capacity of Y.
integer . (We get a value of 4.5). But if n We need to know the relative capacity of buckets X
= 20, then the expression has a value of 2, and Y, and also to what capacity each was filled
an integer. But we have no means of before the transfer. (We have information for Y, but
knowing whether n is 10 or 20, because we need the information for X in this regard.) If we do
both these values are consistent with the not have these two pieces of information, we cannot
specification n < 60. We must conclude answer the question.
Statement 1 tells us that X was filled to some fraction
that statement 1 alone is not sufficient. of its capacity before the transfer. This information
Our choices are B, C or E. alone is not sufficient. We need to know the relative
Statement 2 tells us that n is a multiple of sizes of the two buckets. Our choices are B, C or E.
10. If n is 10, then the expression is not Statement 2 tells us that the proportion information,
an integer. If n is 20, then the expression but we do not know to what capacity X was filled
is an integer. We are getting “all-over- before the transfer. Choices narrow to C or E.
the-map” solution with this information. When we combine the two statements, we have all the
We must conclude that statement 2 alone information we are looking for to answer the
is not sufficient to answer the question. question. We must go with choice C.
When we combine the two statements, we
must still contend with 10 or 20 or 30 or
40 or 50. Unless n is 20, the expression
cannot be an integer. There is no basis
for concluding that n is 20, We must pick
E.

6. If -10 < k < 10 , is k > 0 ?

(1) k2 > 4

(2) 1 - k > 0
K is specified in a certain range of values. The question is: Does k lie to the right of 0 on the number line ? (Is K
positive?). Statement 1 tells us that k > 2 or k < -2. For example, k could be +5 or -7, and both these values will .
the condition specified, viz. K2 > 4. We cannot conclude definitively that k is positive using statement 1 alone. Our
choices are B, C or E.
Statement 2 tells us that 1 - k > 0 or 1 > k.
If k < 1, k could be 0.9 or -5. Both these values are less than 1. But one value is positive, and the other is not. We are
getting “all-over-the-map” kind of solution with this information in statement 2. Our choices narrow to C or E.
When we combine the two statements, we notice that we have two possible range of values from statement 1; k > 2 or
k < -2 Statement 2 tells us that k < 1.
The only range of values that is consistent with both specifications is k < -2. We can conclude using the combined
information that k is not positive. We can answer the question definitively now. We must pick choice C.

46
©Educational Training Services, A Division of MLI Consulting, Inc.
Not to be reproduced without our express written consent
Name of the Registrant: PARTICIPANTS Registration Number: 0098TURBO-2452USA
8. Did Anne pay less than d dollars, including
tax, for her mink coat? 9. Is c > d ?

(1) The price Anne paid for her mink coat was (1) 1 - c/d > -1
(0.9.d), excluding tax.
(2) 0.5 < c/d < 2.0
(2) The tax payable on mink coat sale is 10
percent of the selling price. Statement 1 tells us that
1 - c/d > -1 or - c/d > -2
What do we need to know to be able to Or -c > -2d or c < 2d
answer the question? We need to know We must manipulate the inequality to the form
obtained in order to make sense of the given
the selling price before or after tax, and information. What do we see here? That c < 2d.
the tax rate. Is this good enough to conclude that c > d ?
Statement 1 tells us the selling price Hardly. Because c could be 1.5 d ( in which
before tax. We need to know the rate of case c > d) or c could be 0.5d in which case c
tax as well to be able to answer the < d. We are getting “all-over-the-map” solution
question. Our choices are B, C or E. with this statement. We must move on to
Statement 2 tells us the tax rate, but not examine statement 2. Our choics are B, C or E.
the selling price before or after tax. Statement 2 tells us that 0.5d < c < 2d
Choices narrow to C or E. If C is 0.7d, then c < d. If x is 1.8x, then c > d.
Not a unique solution. Statement 2 alone is not
When we combine the two statements, we
sufficient. Choices narrow to C or E.
have all the information we need to When we combine the two statements, we deal
answer the question. (We have the selling with the inequality 0.5d < c < 2d (because the
price before tax, and the tax rate). We first statement tells us that c < 2d, and the
must pick C. second one tells us that 0.5d < c < 2d). We have
examined this inequality, and concluded that we
cannot get a unique solution. Neither statement
alone or combined is good for a unique solution
or a definitive answer. We must pick E.

47
©Educational Training Services, A Division of MLI Consulting, Inc.
Not to be reproduced without our express written consent
Name of the Registrant: PARTICIPANTS Registration Number: 0098TURBO-2452USA
12. The price per share of stock X increased by
10. In a certain health club, are more than 2/3 10 percent over the same time period that the
of the members females? price per share of stock Y decreased by 10
percent. The reduced price per share of stock Y
(1) The club has exactly 75 female members. was what percent of the original price per share
of stock X ?
(2) The ratio of female to male members is 3:1
(1) The increased price per share of stock X was
We are looking for some “proportion” equal to the original price per share of stock Y.
information here.
Statement 1 tells us the number of female (2) The increase in price per share of stock X
members. We do not know the value for was 10/11 the decrease in the price per share of
total membership, or the number of males. stock Y.
We cannot answer the question using this
information alone. Our choices are B, C or We learn that the price of stock X went from X to
E. 1.1X, and at the same time, the price of stock Y
Statement 2 tells us exactly what we need went from Y to 0.9Y. The question is:
to know. If the ratio of female to male is What is the ratio of 0.9Y to X? We can determine
3:1, then we know that 3/4 of all members the percent information by multiplying this ratio
in the club are females, and 1/4 are females. by 100.
Is 3/4 greater than 2/3? You bet. Can we Statement 1 tells us that 1.1X = Y.
answer the question posed definitively? Or Y/X = 1.1 or 0.9 (Y/X) = 0.99
Certainly. We pick B. We can determine the percent value by multiplying
0.99 by 100. Statement 1 alone is sufficient to
answer the question. Our choices are A or D.
Statement 2 tells us that 0.1X = 10/11 (0.1Y)
Or Y/X = 1.1
We can live with this information, and answer the
11. If x is an integer, is y an integer? question about (0.9Y/X). Statement 2 alone is also
sufficient to answer the question.
(1) The average of x and y is NOT an We must pick D.
integer.

(2) ( x + y ) = 2. ( x - y )

We have to take a “wait and see” attitude, and see what information comes along in statements 1 and 2 about x and y.
Statement 1 tells us that 1/2(X+Y) is not an integer. This information is true if X and Y are odd and even integers
respectively, or the other way around. (The sum of odd and even integers is odd integer, and the odd integer is not
divisible by 2). Or y could be a fraction, and 1/2(X+Y) will not be an integer in this instance also. We cannot
definitively answer whether Y is an integer or not using statement 1 alone. Choices are B, C or E.
Statement 2 tells us that x + y = 2x - 2y or
x = 3y
X is specified as an integer, but y could be 1/3 or 2/3 or 3/3 or 4/3 or any such values so that x will be an integer. As
you can see, y does not have to be an integer so that x will be an integer. On the other hand, y could very well be an
integer so that x = 3y will be an integer. We cannot be sure on the basis of this information. Choices narrow to C or E.
When we combine the two statements, we notice that we have x = 3y and 1/2(x+y) = Not an integer.
Let us plug in 3y for x. We get 1/2 (3y + y) = 2y = Not an integer. If 2y is not an integer, what conclusion can we
draw? That y is not an integer.
We can answer the question by combining the two statements. We must pick C.

48
©Educational Training Services, A Division of MLI Consulting, Inc.
Not to be reproduced without our express written consent
Name of the Registrant: PARTICIPANTS Registration Number: 0098TURBO-2452USA
13. Any decimal that has only a finite number of 15. Is 2n > 100 ?
non-zero digits is a terminating decimal. Examples:
32, 0.78, and 6.087 are three terminating decimals. If
r and s are positive integers, is the ratio r/s a (1) 3n > 100
terminating decimal?
(1) 90 < r < 100 (2) s = 3
We need to know the values for r and s so that we (2) 2n+1 = 128
can answer the question definitively.
Statement 1 tells us that r lies in a certain range. But
this information alone is not sufficient to determine If 2n > 100, then n must be at least 7.
the ratio r/s. Our choices are B, C or E.
Statement 2 gives us the value for s, but is silent Statement 1 tells us that 3n > 100. The only
about r. We must conclude that statement 2 is not determination we can make from this
sufficient alone. C or E. information is that n is at least 5. Is this useful
When we combine the two statements, we notice
information? No. Because, if n is 5, then 25 is 32.
that if r is 93 or 96 or 99, then r/s will be a
terminating decimal. If r is any ohter value in the But if n is 10, then 2n > 100. “At least equal to
range specified for r, then r/s will not be a 5” is not a useful information. We have to
terminating decimal. Since we do not know what reckon with B, C or E.
value r has, we cannot conclude definitively that r/s Statement 2 tells us that 2n+1 = 128 = 27
is a terminating decimal or not. The combined We can determine the true value for n
information is not good for a unique solution or a definitively: n = 6. Once we know that n = 6,
definitive answer. We must pick choice E. we can conclude that 2n = 26 = 64.
We can answer the question definitively that 2n is
14. What is the area of the rectangular region not greater than 100. We must pick B.
of sides l and w ?

(1) l + w = 14

(2) d = 10 , (d is the diagonal)

We need to know the length and the width of a rectangle so that we can determine its area.
Statement 1 tells us that the perimeter is 28. This information is not enough to determine
what values L and W have. For example, L could be 1 and W could b 13, in which case
the area is 13. But if L is 2 and W is 12, then the area is 24. We are not getting a unique
solution here, are we? Our choices are B, C or E.
Statement 2 tells us that the diagonal is 10 units. We cannot determine the area for a
rectangle from its diagonal information. (We can determine the area for a square using the
diagonal length alone). The only determination we can make is that
L2 + W2 = 102
Choices narrow to C or E.
When we combine the two statements, we have two equations, and two variables:
L + W = 14
And L2 + W2 = 102
We can solve for L and W, and determine the area. We can answer the question using the
combined information. We must pick C.

49
©Educational Training Services, A Division of MLI Consulting, Inc.
Not to be reproduced without our express written consent
Name of the Registrant: PARTICIPANTS Registration Number: 0098TURBO-2452USA

16.
17.

YOUR ANSWER: B

X is defined as a PRIME INTEGER THAT


IS A FACTOR OF Y. The question is:
does that factor X repeat twice as factors of
Y? We need to be able to say definitively
one way or the other: Yes, X2 is a factor of
Y or No, X2 is definitely not a factor of Y.

Statement 1 is not sufficient to come up


with a definitive answer because it tells us
nothing about Y. All we can say is that X
is either 2, 3, or 5 but the question was
whether X repeats with a FREQUENCY of YOUR ANSWER: D
2 as a factor of Y? Because we do not
know anything about Y, we cannot tell
whether X occurs JUST ONCE as a factor For a regular Hexagon that is equilateral and
or Twice or more frequently. We must equiangular, the longest diagonal is 2 times the side
eliminate options A and D now. and the shortest diagonal (that forms the base of a
triangle connecting the two sides that make a V) is
Statement 2 tells us that Y = 36. \/3 times side.
Let us PRIME FACTOR OUT 36 as We can see that length is made up of 3 such short
36 = 22 times 32. diagonals and the length is made up of 2 ½ such
Given that X is a PRIME FACTOR OF Y, long diagonals.
we must conclude that X is either 2 or 3, Therefore, length = 5 times X where X is the value
the prime factors of Y. If the question was: for the side of the hexagon, and
what is X, then we must give up and say Width = 3 times \/3 times X.
that we cannot uniquely compute X but the
question is whether X occurs with an Statement 1 tells us that 5X = 5 or X = 1.
exponent of 2. Regardless of whether X is Therefore, width = 3 times SQRT(3)
2 or 3, it does occur with an exponent or Area of the tile is 5 times 3 SQRT(3) = 15 times
frequency of 2. Statement 2 alone is SQRT3 Statement 1 alone is sufficient to answer the
sufficient to logically conclude that X2 is question uniquely. We must stay with options A and
indeed a factor of Y. We must, therefore, D.
choose option B.
Statement 2 tells us that the width = 3\/3 (X) = 3\/3
or X = 1. This, again, leads us to conclude that
length is 5 so that the area is 15 times SQRT(3).

Each statement allows us to uniquely and


independently compute the area of the tile.

We must, therefore, choose option D.

50
©Educational Training Services, A Division of MLI Consulting, Inc.
Not to be reproduced without our express written consent
Name of the Registrant: PARTICIPANTS Registration Number: 0098TURBO-2452USA
18. In a certain office, 60 percent of the
employees are college graduates and 50 percent
of the employees are over forty years old. If 30
percent of those over forty years of age have a
master’s degree, how many employees are
college graduates? 19. Is n an odd integer ?

(1) Exactly 60 employees are over 40 and have (1) (3n + 1) is an odd integer
a master’s degree.
(2) 2n - 1 is an odd integer
(2) There are four times as many college
graduates as there are those over forty with Statement 1 tells us that 3n+1 is an odd integer.
How do we get an odd integer by adding two
master’s degrees.
integers? One of the integers must be odd, and the
If N is the number of employees, then we other even. Since 1 is an odd integer, we must
have: College graduates = 0.6N conclude that 3n must be even integer. Because 3 is
40+ age = 0.5N NOT even, n must be even integer. We can answer
the question definitively using statement 1 alone.
Master’s &
Our choices are A or E.
40+ age = 30% of (40+age value) Statement 2 tells us that 2n - 1 is odd integer. All
= 0.3 ( 0.5N) = 0.15N that we can conclude is that 2n must be even integer
We need to determine the value for N so because 1 is an odd integer. (Even integer
that we can determine what is the value for minus odd integer gives us odd integer). But 2n will
0.6N. be even for all values of n - both odd and even
Statement 1 tells us that 0.15N = 60. values - because the factor 2 will make 2n even
We can determine that N must be 400, and under all circumstances. We conclude that n is a
0.6N must be 240. We can answer the wild card here, and we cannot definitively answer
whether n is odd or even. We must conclude that
question using statement 1 alone. Our
statement 2 alone is not sufficient, but staement 1
choices are A or D. alone is. We must pick A.
Statement 2 does not provide us with new
information. If we set up the information in
statement 2 in mathematical terms, we get:
0.6N = 4 ( 0.15N)
(College grads = 4 times (40+ and Master’s))
We already know this information at the outset,
and we cannot get a value for N using this
information. We must conclude that statement 2
alone is NOT sufficient to answer the question,
but statement 1 alone is.
We must pick choice A.
Can you see how we need to translate the
“verbal” information in the stem to
mathematical terms before we can make sense of
the question?

51
©Educational Training Services, A Division of MLI Consulting, Inc.
Not to be reproduced without our express written consent
Name of the Registrant: PARTICIPANTS Registration Number: 0098TURBO-2452USA

20.

YOUR ANSWER: A

The question is whether the integer whose units digit is the square and
the tens digit is the triangle is definitely divisible by 4 or definitely
not divisible by 4.

We know (or, should know) that in order for an integer to be divisible


by 4, the last two digits must be such that they make up a two-digit
integer that is divisible by 4 or that is a multiple of 4. For example,
the last two digits must form integers such as 00, 04, 08, 12, 16, 20,
24, …. 96.

Statement 1 tells us that the same two digits – triangle as tens and
square as units – are such that they constitute a two digit integer that
is divisible by 4. We must logically conclude that the Triangle-Square
occurring as the last two digits of the integer in the stem integer must
also make that integer divisible by 4.

We must now stay with options A and D, and eliminate B, C, and E.

Statement 2 alone is NOT sufficient because, the triangle and square


could be 4 and 8 so that the stem integer is divisible by 4. They could
also be 3 and 9 so that the stem integer is NOT divisible by 4. As you
can see, we cannot make a decision having logical certainty using
statement 2 alone but statement 1 was definitive.

We must, therefore, choose option A.

52
©Educational Training Services, A Division of MLI Consulting, Inc.
Not to be reproduced without our express written consent
Name of the Registrant: PARTICIPANTS Registration Number: 0098TURBO-2452USA
1. What is the value of | n | ?
DATA SUFFICIENCY 5 ANSWER KEY
AND EXPLANATIONS
(1) n is a negative integer.

Directions: Each of the Data Sufficiency 2


(2) n = 9
problems below consists of a question and two
statements, labeled (1) and (2), in which certain We should remember that the
data are given. You have to decide whether the
data given in the statements are sufficient for absolute value of a number is the
answering the question. Using the data given in distance from the center on a
the statements plus your knowledge of number line, and that distance is
mathematics and everyday facts (such as the always a positive quantity.
number of days in July or the meaning of Statement 1 tells us that n is a
counterclockwise), you are to fill in oval,
negative integer. This information is
A if statement (1) ALONE is sufficient, but not good for a unique solution,
statement (2) alone is not sufficient to answer the because we can think of an endless
question asked; number of negative integer values.
Our choices narrow to B, C or E.
B if statement (2) ALONE is sufficient, but
statement (1) alone is not sufficient to answer the Statement 2 tells us that n = +3 or
question asked; n = -3.
Either way, the absolute value of n is
C if BOTH statements TOGETHER are 3.
sufficient to answer the question asked, but We can determine the precise value
NEITHER statement alone is sufficient.
for the absolute value of n, an answer
D if EACH statement ALONE is sufficient to the question definitively. We
answer the question asked; conclude that statement 2 alone is
sufficient, but statement 1 alone is
E if statement (10 and (2) TOGETHER are NOT not. We must pick choice B.
sufficient to answer the question asked, and
additional data specific to the problem are
needed.

All numbers used are real numbers. A figure in


the data sufficiency problem will conform to the
information given in the question, but will not
necessarily conform to the additional
information given in statements (1) and (2).

All figures lie in a plane unless otherwise


indicated.

In questions that ask for the value of a quantity,


the data given in the statements are sufficient
only when it is possible to determine exactly one
numerical value for the quantity.

53
©Educational Training Services, A Division of MLI Consulting, Inc.
Not to be reproduced without our express written consent
Name of the Registrant: PARTICIPANTS Registration Number: 0098TURBO-2452USA
2. If x, y and z are the three internal angles of 4. How many more men than women are in the
a triangle, what is the value, in degrees, of angle public swimming pool?
x ? (all values of x, y and z are in degrees)
(1) There are twice as many men as there are
(1) x + y = 127 women.

(2) x + y = 2z + 21 (2) If six more women came into the pool, there
will be an equal number of men and women.
We know that x + y + z = 180o. The question
is: What is x ? Statement 1 tells us that the number of men in
Statement 1 tells us that x + y = 127. This the pool is 2N and the number of women is N. If
information is not sufficient for a unique N is 1, then there is just 1 more man in the pool
solution to the value for x. X and y could take than woman. If N is 10, then there are 10 more
a range of values consistent with the men than women in the pool. Not a unique
specification. X could be 30 and y could 97 picture emerging from this statement. We have
or x could 27 and y could 100, and so on. to deal with B, C or E.
Our choices are B, C or E. Statement 2 tells us that if 6 more women came
Statement 2 tells us that x + y = 2z + 21. This into the pool, there will be an equal number of
equation tells us that 180-z = 2z + 21 or z = men andwomen. This can mean only one thing:
159/3 = 53o. When we plug in this value for z There are 6 more men than women in the pool.
in x + y = 2z + 21, we get x+y=127. This Choice B.
information is the same as the one in
statement 1, and is not good for a unique
solution for x. C or E.
When we combine the two statements, we are We get the following picture from the information given
stuck with the same equation twice, and we in the stem:
cannot solve for two variables with just one Number taking the test 0 time(s) = 0.6n
equation. We must pick choice E. Number taking the test 1+ time(s) = 25% (0.4n)
= 0.1n
The question is: How many have taken the test just
once? We know that 0.3n have taken the test just once
3. Of the n people participating in a test previously (Difference of n and (0.6n+0.1n)). We need a
preparation program, 60 percent had not taken number value corresponding to 0.3n.
the test previously. Of the remaining, 25 percent Statement 1 tells us the number corresponding to “more
had taken the test more than once previously. than twice”. We do not have any information given to
How many had taken the test just once us so that we can relate this number to an expression.
previously? We have no use for this statement. Our choices narrow
to B, C or E.
(1) 12 people had taken the test more than twice Statement 2 tells us that 18 people took the test more
previously. than once. We can relate this number to 0.1n and
conclude that n = 18/(0.1) = 180.
(2) 18 people had taken the test more than once We can determine what 0.3n is from this information:
previously. 54 people.
We conclude that statement 2 alone is sufficient, and
pick B as the answer.

54
©Educational Training Services, A Division of MLI Consulting, Inc.
Not to be reproduced without our express written consent
Name of the Registrant: PARTICIPANTS Registration Number: 0098TURBO-2452USA
backwards and conclude that the
numbers in the set must be
31,33,35,37,39, and 41. We can
5. Is the sum of a set of six consecutive odd answer the question one way or the
positive integers a perfect cube? other by stating that “yes, theh sum
is a perfect cube” oor “no, the sum
(1) The smallest number in the set is a prime is NOT a perfect cube”. Why?
number Because we know the precise values
(2) The largest number in the set is 41
in the set. In fact, in this set of
values, the sum is 216, which is a
perfect cube. (63). We conclude that
statement 2 alone is sufficient to
answer the question, and pick B.
A perfect cube is an integer, which Do you understand the type of
can be expressed as the third power reasoning you have to apply to come
of another integer. For example, 27 to a determination of whether the
is a perfect cube because we can information makes sense , or is good
write 27 = 33. (Similarly, a perfect for a unique solution or not. As you
square is an integer that can be might have noticed, we have to
expressed as the square of another invoke our conceptual understanding
integer. 4 isa perfect square because of what a perfect cube is before we
4 = 22). go to deal with the informatin in the
The question is: Does the sum of six statements. You have to pre-
consecutive odd integers add up to a determine what information will
value that can be expressed as a suffice, and then go to examine the
perfect cube? We need to know the information in the statements. .
values in the set so that we can
make this determination.
Statement 1 tells us that the
smallest number in the set of
consecutive odd integers is a prime
number. We could be looking at
3,5,7,9, 11, 13, in which case the
sum is 39, which cannot be
expressed as a perfect cube. On the
other hand, if the numbers were
31,33,35,37,39, and 41, then the
sum is 216, which can be expressed
as 63. As you can see, statement 1 is
not good for a unique solution
because we are getting “all-over-the-
map” solution with the information
provided. Our choices narrow to B, C
or E.
Statement 2 tells us that the last
number in the set is 41. We can work

55
©Educational Training Services, A Division of MLI Consulting, Inc.
Not to be reproduced without our express written consent
Name of the Registrant: PARTICIPANTS Registration Number: 0098TURBO-2452USA
6. How many miles long is the route from
Thunder Bay to Sarnia?
7. If x is divisible by 2, is x + y an even
(1) It will take 2 hours less time to travel the number?
entire route at an average rate of 60 miles per
hour than at an average rate of 50 miles per hour. (1) y is a factor of x

(2) The first half of the distance can be traveled (2) x = 6


in 5 hours at 60 miles per hour rate of
speed. We know that x is even integer. If x+y is to
be even, then y must be even as well. Let us
see if we can get a sense of what y is about
from the statements.
We are dealing with a “distance” problem, Statement 1 tells us that y is a factor of x.
and we must know that distance is a All this means is that we can write x = k.y
function of speed and time of travel. where k collectively represents all the other
Statement 1 tells us that if it took “t” factors. It is possible that k is even and y is
hours to travel the distance at an average odd, in which case x will still be even. It is
speed of 50 mph, then it would take (t-2) also possible that y is even and k is odd,
hours to travel the same distance traveling giving an even value for x. But we are not
at 60 mph. sure which is which, and we conclude that
Our equation is; 50. T = 60 (T-2) statement 1 alone is not sufficient. Our
choices narrow to B, C or E.
We can solve for T: T = 12 hours.
Statement 2 tells us that x = 6. We do not
We can plug this value back into one of know what y is about from this statement.
the equations and conclude that the We conclude that statement 2 alone is not
distance is 50X12 = 600 miles or 60 (12- sufficient.
2) = 600 miles. Statement 1 is good When we combine the two statements, we
enough to answer the question notice that x is 6, and y is a factor of x. We
definitively. Our choices narrow to A or can write 6 as a product of 1 and 6, or of 2
D. and 3. Y could be 1 or 3; or it could be 2
Statement 2 tells us that one half the total or6. In other words, y could be even just as
distance is equal to 5 times 60 = 300 easily as it can be odd. Even after we
combine the two statements, we are not sure
miles. It does not take an I.Q. of one
about y in terms of whether it is even or odd.
trillion to figure out that the total distance We are getting a “may be” kind of a
must be twice 300 miles, or 600 miles. solution, and are not in a position to
Statement 2 alone is also sufficient to conclude definitively that y is even after
answer the question definitively. combining the statements. We must pick E.
Statements 1 and 2 are independently

56
©Educational Training Services, A Division of MLI Consulting, Inc.
Not to be reproduced without our express written consent
Name of the Registrant: PARTICIPANTS Registration Number: 0098TURBO-2452USA

8. All applicants to a military recruitment We have two types of tests, and two possible
program must pass both a written test and a outcomes for each test. We must conclude that it is
physical test. If 60 percent of the applicants a “matrix” problem with which we are dealing.
passed the written test and 75 percent of the Let us set up the matrix using the information in
applicants passed the physical test, what percent the stem:
of the applicants did not pass both or either
tests? WRITTEN WRITTEN TOTAL
PASS FAIL
(1) 55 percent of the applicants passed both tests. PHYSICAL 55%1 X 75%
PASS
PHYSICAL X X 25%
(2) 20 percent of the applicants did not pass FAIL
either test. TOTAL 60% 40% 100%
We need to determine the values in cells marked X
to be able to answer the question.
Statement 1 tells us that 55% passed both tests.
We have marked in the corresponding cell with
the suffix 1 to show that the information is
obtained from statement 1. We can compute the
values for the other remaining cells so that the
column and row totals will agree. We can answer
the question using statement 1 alone. Our choices
narrow to A or D.
Statement 2 tells us that 20% failed both tests. We
have marked this information in the corresponding
cell with the suffix 2, to indicate that this
information was obtained from statement 2.

WRITTEN WRITTEN TOTAL


PASS FAIL
PHYSICAL 55%1 X 75%
PASS
PHYSICAL X 20%2 25%
FAIL
TOTAL 60% 40% 100%
Once again, we can determine all the values in the
cells of the this matrix, and answer the question
definitively using this statement 2 alone.
We conclude that statements 1 and 2 are
independently sufficient to answer the question.
We must pick D.

57
©Educational Training Services, A Division of MLI Consulting, Inc.
Not to be reproduced without our express written consent
Name of the Registrant: PARTICIPANTS Registration Number: 0098TURBO-2452USA

9. If an item was marked up 30 percent on its


cost and then sold at a discount on the sticker 10. If n is an integer, is n/15 an integer ?
price, what was the selling price, in dollars?
(1) 3n/15 is an integer
(1) The item cost $120
(2) 7n/15 is an integer
(2) The sticker price was $156
The question is: is n divisible by 15 or
If the Cost is C, then the sticker price is 15 a factor of n?
was 1.3C. The question is: What is the
selling price after the discount? We Statement 1 tells us that 3n/15 is an
require the discount percentage, and the integer . We can conclude that n/5 is an
cost in dollars to be able to answer the integer or that n is a multiple of 5.
question: n can be any of the following values:
Sticker ( 1- d% ) = Selling price
5, 10, 15, 20, 25, 30, 35, and so on.
100
Unless n is 15 or 30 or 45, or any other
1.3C ( 1- d% ) = Selling price
100 multiple of 15, n/15 cannot be an
Statement 1 tells us what the cost is. integer. Since we do not know what
We can determine that the sticker price value n will have in this range of
was 1.3 times $120. Can we determine possible values, we must conclude that
the selling price using this information statement 1 alone is not sufficient.
alone? We cannot. Why not? Because Choices narrow to B, C or E.
we do not know the discount applied on
the sticker price. Our choices are B, C Statement 2 tells us that 7n/15 is an
or E. integer. Since 7 is a prime integer, and
Statement 2 tells us that the sticker was
not a factor of 15, we must conclude
$156. We can go back and compute the
cost, but we are beating about the bush, that n must be a multiple of 15 so that
and moving in circles not unlike a dog 7n/15 will be an integer. Therefore,
chasing its own tail. We still do not n/15 must be an integer.
have the rate of discount to be abole to
answer the question. Our choices are We must pick choice B.
limitted to C or E.
When we combine the two statements,
we are no better off because we still do
not know what discount rate was
applied on the sticker price. We must
pick E.

58
©Educational Training Services, A Division of MLI Consulting, Inc.
Not to be reproduced without our express written consent
Name of the Registrant: PARTICIPANTS Registration Number: 0098TURBO-2452USA
11. Mary and Martha received wage increases
following their annual performance review? 12. Is John ahead of Paul in the line-up?
Who received the greater increase, in dollars?
(1) There are 20 people in the line-up.
(1) Mary received a 10 percent increase on her
wages. (2) There are exactly 6 people between Paul and
John in the line-up.
(2) Martha received a 8 percent increase on her
wages. Statement 1 does not tells us the relative
We need to know the greatest “dollar value” positions of Paul and John in the line-up. All
increase received by one of the ladies. that we know is that there are 20 people in the
Statement 1 tells us that Mary received 10% line-up. Not good. Choices are B, C or E.
increase on her wages. All that we know is Statement 2 tells us that there are 6 folks
that her wages went from $X to $1.1X. Do we between Paul and John. That does not tell us
get a sense of how many dollars worth of whether Paul is ahead of John or the other way
increase Mary received? Hardly. Besides, we around. Choices narrow to C or E.
have no information pertaining to the other When we combine the two statements, we do
lady in the picture here. We must conclude that not have any information that lets us get a
statement 1 alone is not sufficient. Choices handle on the relative positions of Paul and
narrow to B, C or E, and your odds have John in terms of who is ahead of whom. We
improved significantly , odds of picking the must pick E.
right answer.
Statement 2 tells us Martha’s wages went from 13. Lorraine can drive from her home to a
$Y to $1.08Y. We are clueless in Chicago super-market by one of two possible routes. If
about what Y is all about. We cannot she must also return by one of the two routes,
determine a dollar value for the increase that what is the distance of the shorter route?
Martha received. Statement 2 is silent about
Mary. Our choices are C or E. (1) When Lorraine drives to the supermarket by
When we combine the two statements, we are the shorter route and returns by the longer route,
still not making progress, because we do not she travels a total of 11 miles.
have any information in either statement that
gives us a handle on the dollar values involved (2) When she drives both ways, from her home
in the raises. to the super-market, by the shorter route, she
We must conclude that the statements alone or travels a total distance of 8 miles.
combined are about as useful as a cheap If S is the length of the shorter route, and L that
umbrella during a hurricane onslaught. of the longer route, then we know that
We must pick E. S+L = 11 miles from statement 1. This does
not tell us what the value for S is. As you can
see, you have two variables, and one equation.
Not good enough. Choices narrow to B, C or E.
Statement 2 tells us that S + S = 8 miles or
S = 4 miles. This is the answer we are looking
for. We can answer the question definitively
using statement 2 alone, but statement 1 alone
is not sufficient. We must pick B.

59
©Educational Training Services, A Division of MLI Consulting, Inc.
Not to be reproduced without our express written consent
Name of the Registrant: PARTICIPANTS Registration Number: 0098TURBO-2452USA
P is defined as a prime number having a value
less than 100. Is p equal to 31? Can we
answer this question using the information in
statement 1 or 2 or combined?
14. If p < 100, is the prime number p equal to Statement 1 tells us that p = n3 + 4,where n is
31 ? an integer. The values for n are subject to the
3 constraint that p be a prime integer less than
(1) p = n + 4 , where n is an integer. 100. Can n be 1? If n = 1, then p = 5, a prime
integer. Can n be 2? If n=2, then p = 23 + 4 =
(2) p + 5 is a perfect square. 12, not a prime integer. Therefore, n cannot be
2. Can n be 3 ? if n is 3, then p = 33 + 4 = 31, a
prime integer. Can n be 4? If n is 4, then p = 43
+ 4 = 68, not a prime integer. n cannot be 4.
Can n be 5? If n is 5, then n3 is greater than
100, and will not satisfy the constraint
specified. Therefore, for values of n equal to 1
and 3, p could be 5 or 31. Two possible
values. Not a unique solution. Statement 1 is
not good for a unique solution. We must deal
with choices B, C, or E.
Statement 2 tells us that (p+5) is a perfect
square. A perfect square is an integer that can
be expressed as the square of another integer.
Example: 4 is a perfect square because 4 can
be written as 22.
Knowing that p is a prime integer, the only
values for p that will give a perfect square
value for (p+5) are 11, 31, and 59. If p is 11,
thenp+5 is 16, a perfect square. If p is 31, then
p+5 is 36, a perfect square. If p is 59, then p+5
is 64, a perfect square. Using the information
in statement 2, we conclude that p can have 3
possible values: 11, 31, and 59. Is this a unique
solution? Hardly. What do we conclude? That
statement 2 alone is not sufficient for a unique
solution that lets us answer the question
definitively. Our choices narrow to C or E.
When we combine the two statements, we
notice that statement 1 gives two possible
values for p : 5 and 31
Statement 2 gives us 3 possible values for p:
11, 31, and 59.
Is there a common value in both sets? You
bet. It is 31. When we combine the two
statements, we can conclude that p must be
31 so that this value will satisfy both
conditions stipulated in the two statements.
We must pick choicee C.

60
©Educational Training Services, A Division of MLI Consulting, Inc.
Not to be reproduced without our express written consent
Name of the Registrant: PARTICIPANTS Registration Number: 0098TURBO-2452USA
16. If a television commercial consists of a total
of 17,280 frames on film, how long, in minutes,
does the commercial run?
15. What is the ratio of x to y ?
(1) The commercial runs without interruption at
(1) If the value of y were increased by 7, then the rate of 24 frames per second.
the ratio of x to y will be equal to 1.
(2) It takes 6 times as long to run the film as it
(2) The ratio of x to 4y is 3 to 5. takes to rewind the film, and it takes a total of
14 minutes to do both.
Statement 1 tells us that x = y+7. Can
we determine the ratio in terms of a We need to know the number of frames
specific value for x/y using this per second or per minute that run so
information? No. Because, the that we can determine the time.
equation x = y+7 is true for a multitude Remember, we know already how
of vlaues of y. If y is 1, then x is 8, and many frames the commercial has.
the ratio of x to y is 8/1. If y is 2, then Statement 1 tells us the number of
x is 9, and the ratio of x/y is 9/2. You frames per second that run. We can
are beginning to see “all-over-the-map” determine how long it will take to run
solutions already. We must conclude the commercial with 17,280 frames at
that statement 1 alone is not sufficient. the rate of 24 frames per second. We
We have choices B, C and E left. conclude that statement 1 alone is
Statement 2 tells us exactly what we sufficient, and our choices are A or D.
want to know: if x/4y = 3/5, then x/y Statement 2 states that it takes 6t
= 12/5. minutes to run the film, and t minutes
We have a unique solution, and we can to rewind the film for a total of 7t
answer the question definitively. We minutes, which is equal to 14 minutes.
cannot ask for a better deal than that. We can conclude that it must take 12
We conclude that statement 1 alone is minutes to run the film. Statement 2 is
not good for a unique solution, but also independently sufficient to answer
statement 2 alone IS. We must pick the question.
choice B. We must pick choice D.

61
©Educational Training Services, A Division of MLI Consulting, Inc.
Not to be reproduced without our express written consent
Name of the Registrant: PARTICIPANTS Registration Number: 0098TURBO-2452USA

17. The symbol @ represents one of the 18. If y = 2x+1 , what is the value of y - x ?
following operations: addition, subtraction,
multiplication, or division. What is the value of
4@3? (1) 22x+4 = 64

(1) 0 @ 4 = 4
(2) y = 22x
(2) 4 @ 0 = 4
Statement 1 tells us that 22x+4 = 26
or x = 1. Knowing the value for x,
Statement 1 tells us clearly that @ we can compute what y must be: y
symbol must stand for “addition”. It = 2x+1 = 22 = 4. Statement 1 alone
cannot be “subtraction” because, if it
is sufficient to answer the question.
were, 0 @ 4 will be -4. Multiplication
and Division of 0 by 4 will likewise Our choices narrow to A or D.
yield 0. Therefore @ must stand for Statement 2 tells us that y = 22x.
addition. Statement 1 is good for a When we combine this information
unique solution. Our choices narrow to with the information provided at
A or D. the outset, viz. Y = 2x+1, we can
Statement 2 is not so clear. @ could be
conclude that 2x = x+1 or x = 1.
“addition” or “subtraction” because, in
both cases, 4 @ 0 will be equal to +4. Once again, we have a unique
We conclude that statement 2 alone is value for x, and we can determine
not sufficient, but statement 1 alone is what y = 2x+1 must be. It is equal
sufficient. We must pick A. to 4.

Statement 2 also is independently


sufficient to answer the question
definitively.

We must pick D.

62
©Educational Training Services, A Division of MLI Consulting, Inc.
Not to be reproduced without our express written consent
Name of the Registrant: PARTICIPANTS Registration Number: 0098TURBO-2452USA
20. If x , y , and z are three integers, are they
19. In a certain group of people, the average consecutive odd integers?
(arithmetic mean) I.Q of the males is 128 and
of the females 136. What is the average I.Q. of (1) y - x = 2
the people in the group?
(2) x + y is even
(1) The group contains twice as many females
as males We need information about x, y, and z to
answer the question. We notice that
(2) The group contains 10 more females than statements 1 and 2 do not provide any
males. information about z. We cannot answer the
This is a weighted average problem. question using the statements 1 and 2
Statement 1 gives us the proportion of men and independently, or combined. We pick E.
women in the group. We can work with this
information. How? If N is the number of
males, then 2N is the number of females. 21. In triangle ABC, if AB = x, BC = x + 2,
What is the average I.Q. for the group? and AC = y, then which of the three internal
Average = 128N + 136(2N) = 400N = 133 angles of the triangle has the greatest degree
N+2N 3N measure?
As you can see, statement 1 is good for a
unique solution, and a definitive answer. You (1) y = x + 3
must set up the problem in the above manner
before concluding whether the information is (2) x = 2
good for a solution or not.
Our choices narrow to A or D. The angle opposite the longest side will
Statement 2 tells us that F = M + 10 have the largest measure.
If N is the number of males, then F = N + 10.
Statement 1 tells us that AC is the
Let us see what happens when we try to set up
the equation for average: longest side, because x+3 is the largest
Average = N(128) + 136(N+10) of the three values for the sides of the
N + N + 10 triangles. We can conclude that the
= 264N + 1360 angle opposite AC must be the largest
2N + 10 angle in the triangle. Statement 1 alone
As you can see, we cannot get a number
is sufficient. A or D.
value for this expression. We must
Statement 2 is not so precise. We have
conclude that statement 2 is not good for
a unique solution. Rememeber; you some value for x, but we are clueless
must set up the problem in the manner about y. We cannot answer the question
suggested before concluding that the definitively using the information in
information is useless. statement 2 alone.
We must pick A, because statement 1 We conclude that statement 2 alone is
alone is sufficient, but statement 2 alone not sufficient, but statement 1 alone is
is NOT. sufficient. We pick A.

63
©Educational Training Services, A Division of MLI Consulting, Inc.
Not to be reproduced without our express written consent
Name of the Registrant: PARTICIPANTS Registration Number: 0098TURBO-2452USA

DATA SUFFICIENCY 6 EXPLAINED 2. In an election to the Secretary of the Club, if


each of the 1,000 members voted for either Mary
1. Is x greater than 2? or Michelle (but not both), what percent of the
female members voted for Michelle?
2
(1) x > 4
(1) Eighty percent of male members voted for
(2) x is a multiple of 2 Michelle.

Your Answer; A ( ) B ( ) C ( ) D ( ) E ( ) (2) Twice as many male members voted for


Í We are required to compare X and 2 and Michelle as female members.
decide which is bigger. This is how you must
understand the question posed. Your Answer; A ( ) B ( ) C ( ) D ( ) E ( )
Statement 1 tells us that X > 2 or X < -2. This is
how you must interpret X2 > 4. Under the first Í Our “Need to Know” information is: How
likely scenario, X is indeed more than 2. Under many females there are in the group, and how
the second equally likely scenario, X is not more many of those females voted for Michelle.
than 2. We have a conflict here and cannot make
a decision in the face of conflict. We must Statement 1 tells us that 80% of males voted for
eliminate options A and D, and work with the Michelle. This information does not provide any
three remaining options B, C, E. of the two pieces of information we set to seek
Statement 2 tells us that X is a multiple of 2. out in our “N.T.K”. We have to eliminate
Likely values for X are: options A and D because Statement 1 is not good
-4, -2, 0,2,4,6,………….. enough to make a decision with.
This is how you must read “multiple of 2” We have B, C, and E left to choose from.
statement. As you can see, some of the likely
values are more than 2, others are equal to 2, and Statement 2 tells us that
still others are less than 2. We have another Mmichelle voters = 2 FMichelle voters
major conflict here, and conflict is not good for a
definite yes or no decision. We must eliminate This ratio information does not tell us any of the
option B and work with the remaining C and E. two pieces of information required in our N.T.K
If we combine the two statements, we notice that to answer the question posed. We cannot work
–2, 0, and 2 are not permissible values for X but with statement 2 alone also. We have to
the other permissible values satisfying both eliminate option B, and move to combine the
statements give rise to a conflict: some are more two statements.
than 2 and others are less than –2. Once again, When we combine the two statements, we notice
we cannot answer the question posed with a that the Females who voted for Michelle
unique yes or no decision. We must pick option represent 40% of the Male members in the club.
E. But then, if the males represent 50% of the
Remember: If X2 > 4, then X > 2 or X < -2. group, then the females voting for Michelle are
IF X2 < 4, then X < 2 and X > -2. In other words, 40% of 50% = 20% of the group. If the males
-2 < X < 2. represent 70% of the group, then the females
voting for Michelle are 40% of the 70% = 28%
of the group. This conflict is bad enough. We are
also unable to answer the question posed: What
percent of Females voted for Michelle. We have
to pick Answer choice E.

64
©Educational Training Services, A Division of MLI Consulting, Inc.
Not to be reproduced without our express written consent
Name of the Registrant: PARTICIPANTS Registration Number: 0098TURBO-2452USA

3. During 1987, 8.7 percent of the men in the 4. If the average (arithmetic mean) of 4
labor force were unemployed in June as against numbers is 45, how many numbers are greater
8.4 percent in May of that year. If the number of than 45?
men in the labor force was the same in both
months, how many more men were unemployed (1) Two of the numbers are 60 and 45.
in June than in May?
(2) One of the numbers is 25.
(1) The number of unemployed men in the labor
force during May was 1.68 million. Your Answer; A ( ) B ( ) C ( ) D ( ) E ( )

(2) The total number of men in the labor force Í We are told that the sum of the four numbers
was 20.0 million during the two months - May is 180. We are required to make a decision about
and June - of 1987. how many numbers are greater than 45. Notice
that the problem talks about “numbers”, not
Your Answer; A ( ) B ( ) C ( ) D ( ) E ( ) about “integers”.

Í If X represents the number of men in the Statement 1 tells us that the first two numbers
labor force, then we are required to find out the add up to 105. The remaining two numbers must
difference between 0.087X and 0.084X or the add up to 75. Can we tell how many numbers are
value for 0.03X. We will be able to answer the greater than 45? May be one of them is if the
question posed if we can glean some information two numbers are 51.5 and 23.5, or NONE if the
about the value for X. This is our N.T.K. numbers are 37 and 38. There is a conflict and
we cannot make a unique decision. WE must
Statement 1 tells us that 0.084X = 1.68 Million. conclude that statement 1 is not sufficient to
We can determine that X = 20 million, and we make a unique decision, and eliminate options A
can determine what 0.03 of X is using this and D.
information. Statement 1 alone is sufficient to
answer the question posed. We must keep Statement 2 tells us that one of the four numbers
options A and D, and eliminate options B, C, and is 25. All that we know is that the other three
E. If statement 2 is also sufficient, we will pick numbers add up to 155. Can we tell how many
D. Otherwise, we will go with A. of these three remaining numbers are more than
45? We cannot. May be all three are greater than
Statement 2 is even better. It tells us that X = 20 45 if the numbers happen to be 50, 51, 54. Or,
million. We can compute what 0.03X is using just two are more than 45 if the numbers happen
this information. to be 60, 50, and 45. Can you see the conflict
here? We cannot make a unique decision by
We notice that statements 1 and 2 are using statement 2 also. We must eliminate option
independently sufficient to make a decision. We B and move on to combine the two statements.
must choose option D.
When we combine the two statements, we notice
Note that your ability to predetermine your that we know what the three of the four numbers
“Need to Know” information, and to process a are, and we can determine what the fourth
lot of verbiage into simple algebraic terms is a number is and answer the question in a unique
critical ability in this part of the GMAT. fashion. The fourth number must be 50, and
given that another one is 60, there are two
numbers greater than 45. We must choose option
C.

65
©Educational Training Services, A Division of MLI Consulting, Inc.
Not to be reproduced without our express written consent
Name of the Registrant: PARTICIPANTS Registration Number: 0098TURBO-2452USA

5. Is the value of x greater than 5? Because we get two likely values for N + 1, +7
and – 7, we cannot make a unique decision about
(1) x3 > 125 whether N + 1 is prime or not. If N+1 is 7, then
it is prime. If N+1 is –7, it is not prime. We
(2) 4 < x < 6 must eliminate option B, and move on to
combine the two statements.
Your Answer; A ( ) B ( ) C ( ) D ( ) E ( ) When we combine the two statements, we notice
that N + 1 must be 7 because the product two
Í We are required to compare X and 5, and prime numbers cannot be negative, and N + 1
decide in a unique fashion which is better. This cannot be a negative value (because N cannot be
is how you must understand the question posed. negative according to statement 1).

Statement 1 tells us that X > 5. We can tell that When we combine the two statements, we can
X is indeed greater than 5 because that is what make a decision that N + 1 is indeed prime. We
this statement leads us to believe. We must keep must pick option C.
options A and D, and eliminate options B, C, and
E. 7. A rectangular frame encloses a picture. What
is the length in inches of the picture?
Statement 2 is not good for a unique decision. X
could be 4.5 or 5.9. WE cannot decide whether (1) The frame measures 24 inches by 18 inches.
X is definitely more than 5 or less than 5. We
must conclude that statement 2 alone is not (2) Area of the frame = area of the picture it
sufficient to make a decision. encloses.

We must pick answer A. Your Answer; A ( ) B ( ) C ( ) D ( ) E ( )

Í We need to look for information allowing us


6. Is n + 1 a prime number? to compute the length of the picture in a unique
way.
(1) n is a product of two prime numbers. Statement 1 tells us about the frame size. We are
not able to determine the length of the picture by
(2) n2 + 2n + 1 = 49 knowing the size of the frame. We cannot make
a decision by using statement 1 alone. Let us
Your Answer; A ( ) B ( ) C ( ) D ( ) E ( ) eliminate options A and D.
Statement 2 says that the area of the frame is the
Í We need to know what N is so that we can same as the area of the picture it encloses.
determine what N+1 ought to be. Obviously the total frame area is divided in two
Statement 1 tells us that N could be 6, which is equal halves, but we are not able to determine
the product of 2 and 3 – both prime integers -, what the picture area is on the basis of this
and in this scenario N + 1 is 7, a prime number. statement alone. Let us eliminate choice B.
But then, N could also be 15 – product of 3 and When we combine the two statements, we notice
5, both prime integers – and N + 1 is 16, which that the picture area is ½ of 24 X 18 square
is not a prime number. Can you see the conflict inches. But we cannot determine the length if we
here? We cannot make a unique decision by know the picture area unless we know what the
using statement 1 alone. We must eliminate width is or unless the problem tells us that the
options A and D, and keep options B, C, and E picture is a square shaped one. We tried all the
only. tricks in the book, and still cannot make a unique
decision about the length of the picture. We must
Statement 2 tells us that (N+1)2 = 49 or N + 1 = go with option E.
7 or N + 1 = -7. Of course, you were required to
recognize that N2 + 2N + 1 is the same as (N +
1)2. “Recognition” is an essential skill you will
require if you want to do well in the GMAT.

66
©Educational Training Services, A Division of MLI Consulting, Inc.
Not to be reproduced without our express written consent
Name of the Registrant: PARTICIPANTS Registration Number: 0098TURBO-2452USA
8. How far apart do Jane and June live?

(1) The local public library is 5 miles due north


of Jane’s house and 12 miles due east of June’s
house. 9. If n > 0, is n equal to the sum of
two different prime numbers?
(2) The local school is 12 miles due west of
Jane’s house and 5 miles due south of June’s (1) n is equal to the square of the smallest odd
house. prime number.

Your Answer; A ( ) B ( ) C ( ) D ( ) E ( ) (2) n + 2 is a prime number.


Í Statement 1 gives us the following picture.
June 12 Library Your Answer; A ( ) B ( ) C ( ) D ( ) E ( )
Í The question is: Can we determine whether
5 we can write N as the sum of two different prime
integers? In order to answer this question, we
Jane need to know what N is.
We get a 5:12:13 triangle here, and Jane and Statement 1 tells us that N is equal to 32 = 9. We
June must live 13 miles apart. We must keep can write 9 as the sum of 2 and 7. Yes, N can be
options A and D, and eliminate choices B, C, written as the sum of two different prime
and E. integers. Statement 1 allows us to make a unique
decision. We will keep options A and D, and kill
Statement 2 gives us the following picture: options B, C, and E.
Statement 2 is not as precise as statement 1 was.
JUNE If N+2 is a prime integer, N could be 3 and N+2
5 could be 5, a prime integer. But 3 cannot be
written as the sum of two different prime digits.
SCHOOL 12 JANE But then N could be 5 and N + 2 is 7, another
prime integer. Here, we can write 5 as the sum of
Once again, we see a 5:12:13 triangle, and can two different prime integers – 2 and 3. We are
determine that June and Jane live 13 miles apart. dealing with two different conflicting scenarios
now allowing us to make a unique decision.
Each statement independently allows us to We will pick option A.
answer the question posed in a unique sort of
way. We must choose option D.
Remember to recognize the standard triple
proportions when you deal with the righ
triangles: 3:4:5 and 5:12:13, and occasionally
7:24:25.

67
©Educational Training Services, A Division of MLI Consulting, Inc.
Not to be reproduced without our express written consent
Name of the Registrant: PARTICIPANTS Registration Number: 0098TURBO-2452USA

10. 2/3 of the N people polled in a survey


involving 2 questions said Yes to Question 1.
What fraction of the people polled did NOT say
yes to both questions?
11. If s, u, and v are positive integers, is
(1) 3/5 of those who answered YES to Question s > v?
1, answered YES to Question 2.
(1) s > u
(2) 1/3 of those polled answered NO to Question
1. s u v
(2) 2 = 2 + 2
Your Answer; A ( ) B ( ) C ( ) D ( ) E ( )
Your Answer; A ( ) B ( ) C ( ) D ( ) E ( )
Í We have two questions, and each question Í WE are required to compare S and V, and
has two mutually exclusive response decide which is bigger.
possibilities. We can represent this scenario in
terms of a table as follows. According to the Statement 1 tells us that there is a connection
question stem, the total for Q1 YES is 2N/3. between S and U. We cannot use this
This tells us that the number answering NO to information to arrive at any conclusion about S
Q1 is 1/3 N. and V. We must eliminate options A and D.
Q1 YES Q1 NO TOTAL Statement 2 tells us that S = V+1 and S = U+1.
If S =2, then U = 1 and V = 1.
Q2 YES 2N/5(ST.1) If S = 3, then U = 2, and V = 2.
If S = 4, then U = 3, and V = 3 and so on.
Q2 NO 4N/15(ST.2) ?????? This statement clearly tells us that S is bigger
(stem) than V.
TOTAL 2N/3 N/3(stem) N
We can make a unique decision by using
statement 2 alone but not by using statement 1
According to statement 1, 3/5th of 2N/3 answered alone. We must pick option B.
YES to Q2. This gives us 2/5th of N for Q1 YES
and Q2YES. This information allows us to 12. What is the area of the square region?
compute the value for those answering Yes to
Q1 but NO to Q2. (2N/3 – 2N/5= 4N/15). We (1) The diagonal is 10 inches.
cannot
Compute the value for Q1NO and Q2 NO cell on (2) The perimeter is 20 \/2
the basis of statement 1 information alone. We
must eliminate options A and D. Your Answer; A ( ) B ( ) C ( ) D ( ) E ( )
Statement 2 does not tell us anything new. If the
stem tells us that 2/3rd of N people answered Yes Í We can compute the area of a square in terms
to Q1, by default we know that 1/3rd of N of its side (side2) or in terms of its diagonal
answered NO to Q1. We cannot determine the (diagonal2 / 2).
value for the shaded cell on the basis of this Statement 1 tells us what the diagonal is. We
statement alone. know that the area must be 102/2 = 50. We can
Even if we combine the two statements, we are make a unique decision by using statement 1
not looking at any new piece of information alone. Let us keep options A and D, and
allowing us to compute the value for the shaded eliminate options B, C, and E.
cell shown in the table above. Statement 2 tells us that the side of the square is
We must pick option E. 1/4th of the perimeter or 5\/2. We can square the
side and get the area using statement 2 alone
also.
We must choose option D.

68
©Educational Training Services, A Division of MLI Consulting, Inc.
Not to be reproduced without our express written consent
Name of the Registrant: PARTICIPANTS Registration Number: 0098TURBO-2452USA

13. What is the measure of an external angle of


the triangle ABC? 15. What is the value of k?

(1) One of the internal angles measures 72 (1) In the xy-coordinate system, (a,b) and (a+3,
degrees. b+k) are two points that lie on the line defined
by the equation x = 3y - 7.
(2) The triangle ABC is an isosceles triangle.
(2) k.k = 1
Your Answer; A ( ) B ( ) C ( ) D ( ) E ( ) Your Answer; A ( ) B ( ) C ( ) D ( ) E ( )
Í We are asked whether we can determine the
value for “any one” of the external angles of a Í Statement 1 allows us to compute the slope of
triangle. We should know that the external angle the line defined by the equation x = 3y – 7 by
is the adjacent angle to an internal angle. rearranging the equation in the standard form:
Statement 1 tells us that one of the internal Y = 1/3 X + 7/3. We can see that the slope is 1/3.
angles is 72 degrees. We know that the We can also compute the slope by using the two
corresponding external angle is 180 – 72 = 108 points (a, b) and (a+3, b+k) as
degrees. We must keep options A and D, and Slope = (b – b –k)/(a – a –3) = k/3
eliminate options B, C, and E. We know that the two expressions for the slope
Statement 2 tells us that two angles are equal in are identical and we can set them equal:
the given triangle. But we do not know what the K/3 = 1/3 or K = 1. This is a unique value for K
angles are, and cannot determine the and statement 1 alone is sufficient. Let us keep
corresponding external angle. options A and D.
We must pick option A. Statement 2 tells us that K = 1 or K = -1.
Not a unique value for K. We cannot make a
decision by using statement 2 alone whereas
14. What is the ratio of the volume of cube X to statement 1 alone was sufficient.
that of cube Y? We must pick option A.

(1) The length of an edge of cube X is 6 inches.

(2) The ratio of the surface area of cube X to that


of cube Y is ¼.

Your Answer; A ( ) B ( ) C ( ) D ( ) E ( )

Í The volume of a cube is a function of its


side3. If X is the side of cube X and Y that of
cube Y, we are required to compute a unique
value for the ratio X3/Y3.

Statement 1 does not allow us to compute this


ratio because it does not tell us anything about
cube Y. We must eliminate options A and D.
Statement 2 tells us that 6X2/6Y2 = ¼ or
X2/Y2 = ¼ or X/Y = ½.
IF we cube (X/Y) we get the ratio of the volume
of X to that of Y. We can detemine that the ratio
of volume of X to that of Y is (1/2)3 = 1/8.
Statement 2 allows us to get a unique value for
the ratio, whereas statement 1 does not. We must
pick option B.

69
©Educational Training Services, A Division of MLI Consulting, Inc.
Not to be reproduced without our express written consent
Name of the Registrant: PARTICIPANTS Registration Number: 0098TURBO-2452USA
16. What is the average (arithmetic mean) dollar When we combine the two statements, knowing
amount of all the paychecks that John Doe TB = 5, and knowing that TATB/(TA+TB) = 2, we
received last year? can determine TA.
The combined information is good for
(1) Last year John Doe received 26 paychecks. making a unique decision but each
statement independently was not.
(2) The average of John Doe’s first We must pick option C.
thirteen checks during the year was
$750. The average of John Doe’s last 18. How many students are in the school?
13 checks was $800.
(1) 40 more than 1/3 of all the students in the
Your Answer; A ( ) B ( ) C ( ) D ( ) E ( ) school are taking a science course and, of these,
Í We need to know the total value of his paychecks and 1/4 are taking Physics.
how many he received during the year so that we can
compute the average value.
(2) Exactly 1/8 of all the students in the school
Statement 1 tells us that John received 26 paychecks but we are taking Physics.
do not know the total value of those checks. We cannot use
statement 1 alone to make a decision. Let us kill options A Your Answer; A ( ) B ( ) C ( ) D ( ) E ( )
and D. Í If N is the number of students, we are
Statement 2 tells us that the total of his first 13 checks was 13
times 750, and the total of his last 13 checks was 13 times required to verify whether it is possible to
800. WE can compute the total of 26 paychecks but we are determine N.
not going to assume that John did not receive another 5 Statement 1 tells us that
checks in between the first 13 and the last 13. Remember: No Number taking Science = N/3 + 40
unwarranted assumptions in GMAT. Reluctantly, we must
conclude that statement 2 does not allow us to make a unique And Number taking Physics = ¼(N/3 + 40)
decision. Let us eliminate option B. Or, Physics number = N/12 + 10
By combining the two statements, we know that John WE cannot compute N on the basis of the above
received exactly 26 checks, and using statement 2 we can information alone. Let us kill A and D. You
determine the total value of those checks so that we can
determine the average value of those checks. must bear in mind that you need to set up the
Choice C corresponds to this scenario. stated information along the above lines before
you give up.
17. How many hours does it take for Machine A Statement 2 tells us that the number taking
to fill a production lot, working alone at a physics = N/8.
constant rate? Once again, we cannot determine N on the basis
of this statement alone.
(1) Machines A and B, operating simultaneously Let us kill option B.
at their respective constant rates, can fill the When we combine the two statements, we can
production lot in 2 hours. set up the following equation:
N/8 = N/12 + 10.
(2) Machine B, working alone at its constant We notice that we area dealing with a single
rate, can fill the lot in 5 hours. variable equation that allows us to determine a
unique value for N.
Your Answer; A ( ) B ( ) C ( ) D ( ) E ( ) We must pick option C.
Note: Be sure to set up equations before
Í Statement 1 tells us that TATB/(TA+TB) = 2. deciding whether you can make a decision or
We cannot determine either TA or TB. We must not. Data sufficiency requires that you do
conclude that statement 1 does not allow us to minimal implementation before giving up. Do
determine the time Machine A will take to do the not attempt to decide whether you can make a
job working alone at its constant rate. decision or not by simply reading the statements.
Let us kill options A and D.
Statement 2 tells us that TB = 5. We cannot
determine TA by using this information alone.
Let us kill option B.

70
©Educational Training Services, A Division of MLI Consulting, Inc.
Not to be reproduced without our express written consent
Name of the Registrant: PARTICIPANTS Registration Number: 0098TURBO-2452USA
19. Is c2 + d2 > 1? 20. @, &, and * are three different positive
digits and If @ + & = * , What is the value
(1) d > 0 of &?

(2) c/d > 1 (1) * = 4

Your Answer; A ( ) B ( ) C ( ) D ( ) E ( ) (2) @ = 1


Í Statement 1 is not sufficient to answer the
question because we do not know what C is Your Answer; A ( ) B ( ) C ( ) D ( ) E ( )
about. Let us eliminate options A and D.
Statement 2 tells us that C > D. This is not Í The two most important questions that will
sufficient information because if c = ½ and D = stand you in good stead when you take the test
¼, then C2 + D2 = 5/16, not more than 1. are:
But if C = 2 and D = 1, then the ♦ What do I know?
expression is more than 1. We have ♦ What do I need to know?
In this case, we are dealing with three positive
conflicting scenarios consistent with the
digits. We know that the positive digits could be
statement 2 and cannot make a unique any value from 1 through 9. We are asked
decision. We must eliminate option B. whether we can determine a unique value for the
Even when we combine the two statements, we & digit on the basis of information in statements
cannot make a unique decision because anything 1 and 2.
greater than 0 could be a fraction less than 1, and Statement 1 tells us that the sum of @ and & is 4
the first scenario might as well play out as could because the stem tells us that the value for * is
the second scenario discussed above. the same as the sum of the values of @ and &.
We must choose option E.
Our reasoning will establish that @ and & could
be any combinations of 1 and 3, or 3 and 1. This
means that & could be 1 or 3. Notice that 2 and
2 is not a good scenario because the digits are
“different”. The test is going to ask you to show
that you are paying attention.

We notice that the first statement gives us 2


possible values for &, and that is not a unique
situation. We must conclude that statement 1 is
not sufficient to make a decision. Let us
eliminate options A and D.

Statement 2 tells us that the other digit @ is 1.


We cannot determine a unique value for &
because & could be 2 through 8. Notice that the
limiting value for * is 9, and if @ is 1, then &
could be any value 2 through 8. Once again,
there are 7 different likely values for &, and we
cannot work with statement 2 alone also. Let us
eliminate option B.

When we combine the two statements, we know


that @ is 1, and, consequently, & must be 3. The
combined information lets us determine a unique
value for &. We must choose option C.

71

You might also like